Fileshare.ro_the Complete Hematology Guide

October 18, 2017 | Author: maurice_ejw | Category: Anemia, Sickle Cell Disease, Hemoglobin, White Blood Cell, Red Blood Cell
Share Embed Donate


Short Description

ghid de hematologie...

Description

About this guide If you’re reading this introduction, it means you are probably either a) covering hematopathology in your pathology class right now, or b) studying for boards. Either way, you’ve come to the right study guide! Inside, you’ll find a comprehensive (but not oppressive) review of both benign and malignant hematopathology, neatly summarized and nicely illustrated. Whether you have a readthe-text-straight-through kind of mind, or a looking-at-pictures mind, or a question-working mind, you’ll find it easy to work your way through this guide. Extra help If you are stuck, or frustrated, or if something just doesn’t make sense, feel free to email me at [email protected]. I’ll do my best to get you unstuck and back on track.

© 2011 Pathology Student

www.pathologystudent.com

The Complete (but not obsessive) Hematopathology Guide page 2

Table of Contents 1. Introduction… … … … … … … … … … … … … … p. 5 2. Anemia… … … … … … … … … … … … … … … … p. 12 3. Benign leukocytoses… … … … … … … … … .. p. 39 4. Leukemia… … … … … … … … … … … … … … … p. 46 Acute myeloid leukemia………………….. p. 48 Myelodysplastic syndromes……………… p. 59 Acute lymphoblastic leukemia…………… p. 60 Chronic myeloproliferative disorders…….. p. 65 Chronic lymphoproliferative disorders…… p. 72 5. Myeloma… … … … … … … … … … … … … … … .. p. 78 6. Lymph node disorders… … … … … … … … … . p. 80 Benign lymph node disorders……………. p. 80 Non-Hodgkin lymphoma…………………. p. 83 Hodgkin disease………………………….. p. 93 7. Reference section… … … … … … … … … … … . p. 96 8. Study questions… … … … … … … … … … … … . p. 102

www.pathologystudent.com

The Complete (but not obsessive) Hematopathology Guide page 3

List of Diseases Anem ia Iron-deficiency anemia Megaloblastic anemia Hemolytic anemias Hereditary spherocytosis G6PD deficiency Sickle cell anemia Thalassemia Warm autoimmune hemolytic anemia Cold autoimmune hemolytic anemia Microangiopathic hemolytic anemia Anemia of chronic disease Anemia of chronic renal disease Anemia of chronic liver disease Aplastic anemia

Benign leukocytoses Benign neutrophilia Benign lymphocytosis Other leukocytoses

www.pathologystudent.com

Leukem ia

Myelom a

Acute myeloid leukemia AML with genetic abnormalities AML with FLT3 mutation AML with multilineage dysplasia AML, therapy-related AML, not otherwise classified Myelodysplastic syndromes Acute lymphoblastic leukemia T-cell ALL B-cell precursor ALL Burkitt leukemia Chronic myeloproliferative disorders Chronic myeloid leukemia Chronic myelofibrosis Polycythemia vera Essential thrombocythemia Chronic lymphoproliferative disorders Chronic lymphocytic leukemia Hairy cell leukemia Prolymphocytic leukemia Large granular lymphocyte leukemia

Lym ph node disorders Benign lymph node disorders Non-Hodgkin lymphoma Small lymphocytic lymphoma Marginal zone lymphoma Mantle cell lymphoma Follicular lymphoma Mycosis fungoides/Sézary syndrome Diffuse large B-cell lymphoma Lymphoblastic lymphoma Burkitt lymphoma Adult T-cell leukemia/lymphoma Hodgkin disease Nodular lymphocyte predominance Nodular sclerosis Mixed cellularity Lymphocyte rich Lymphocyte depletion

The Complete (but not obsessive) Hematopathology Guide page 4

One. Introduction Clinical stuff Patients with hematologic disorders have lots of widely-varying signs and symptoms! Here’s a short list of things to look for in the history and physical. More details will be discussed as we go through individual disorders. Physical exam

History

Examine

Look for

Might mean

Ask about

Might mean

Sclerae

Jaundice Petechiae

Hemolysis Bleeding disorder

Infections (repeated or unusual)

Leukemia, lymphoma

Tongue

Beefiness Smooth surface

Megaloblastic anemia Iron-deficiency anemia

Bleeding (epistaxis, bleeding gums, menorrhagia, hemarthrosis)

Thrombocytopenia, leukemia, bleeding disorder

Dyspnea, chest pain

Anemia

Skin

Pallor Jaundice Petechiae

Anemia Hemolysis Bleeding disorder

Pica

Iron-deficiency anemia

Abdominal fullness/early satiety

Splenomegaly

Nails

Spoon shape

Iron-deficiency anemia

Alcohol use (excessive)

Megaloblastic anemia

Heart

Tachycardia

Severe anemia

Headache, neurologic deficits

Leukostasis, thrombosis

Nodes

Enlargement

Infection or lymphoma

Pruritis

Polycythemia

Bones

Tenderness

Marrow expansion Multiple myeloma

Prior malignancies, chemotherapy

Secondary malignancies

History of thrombosis

Factor V Leiden

Spleen, liver

Enlargement

Leukemia, lymphoma

Family history of bleeding disorder

Nerves

Decreased vibratory sense

Megaloblastic anemia

Hemophilia, von Willebrand disease

Family history of anemia

Hemoglobinopathy, thalassemia

www.pathologystudent.com

The Complete (but not obsessive) Hematopathology Guide page 5

The Complete Blood Count (CBC) The CBC is comprised of a bunch of different indices. You get all of these on every report, whether you ask for them specifically or not (that’s just the way the machine does it!). Some of these indices are really useful (like the hemoglobin, MCV and RDW), and some of them are rarely if ever used (like the mean platelet volume). You should know what each one measures, and be able to recognize the normal range. Red blood cell count (RBC) • Total number of red blood cells in blood • Normal ranges: male 4.5-6.0 x 1012/L, female 3.8-5.2 x 1012/L Hemoglobin (Hgb) • Concentration of hemoglobin in blood • Normal ranges: male 13-18 g/dL; female 12-16 g/dL • Hgb below normal = anemia

The most useful red cell indices are the hemoglobin, MCV and RDW.

Hematocrit (Hct) • Volume of “packed” red blood cells. • In the old days, was performed by spinning a tube of blood and estimating the amount of total blood volume taken up by the red cells (not such a great method – because if the cells are of unusual shape, they may not pack as well as normal red cells, producing an artificially elevated Hct) • Now calculated by machine (MCV x RBC) • Normal ranges: male 40-52%, female 35-47% Mean red blood cell volum e (MCV) • Average size of red blood cells • Normal range: 80-100 fL (1 fL = 10-15 L) • Differentiates between microcytic (MCV < 80), normocytic (MCV 80-100) and macrocytic (MCV > 100) anemias

www.pathologystudent.com

The Complete (but not obsessive) Hematopathology Guide page 6

Mean cell hemoglobin (M CH) • Weight of Hgb in the average red blood cell • Normal range: 26-34 pg (1 pg = 10-12 g) • Not a frequently used parameter Mean cell Hgb concentration (M CHC) • Concentration of Hgb in the average red blood cell • Normal range: 32-36 g/dL • Calculated by machine (Hgb/Hct) • Used to differentiate between hypochromic (MCHC < 32) and normochromic (MCHC 3236) anemias • There is no such thing as a hyperchromic red cell. • You can see this nicely on a blood smear: normochromic cells have a “zone of central pallor” (that white dot in the middle of the cell) that is no more than 1/3 the diameter of the red cell. Hypochromic red cells have just a thin rim of hemoglobin. Red cell distribution width (RDW ) • Standard deviation of the MCV • Tells you how much the red blood cells differ from each other in size. If they are all pretty similar in size, the RDW is low. If some are little and some are big, the RDW is high. • Normal range = 12-13.5% • Used to differentiate between anemias with minimal anisocytosis (difference in cell size) (RDW 12-13.5%) and those with increased anisocytosis (RDW > 13.5%). • You can see this on a blood smear: when anisocytosis is increased, you’ll see a range of cell sizes – some are smaller, some are bigger. W hite blood cell count (W BC) • Total number of leukocytes in blood • Normal ranges: adult: 4.5-11 x 109/L, newborn: 9 -30, child over 1: 5.0-17.0 • A high WBC is seen in many conditions. Some are benign, such as infection and inflammation (see the Benign Leukocytosis section of this book). Others are malignant, such as leukemia (see the Leukemia section of this book).

www.pathologystudent.com

The Complete (but not obsessive) Hematopathology Guide page 7

Differential (“diff”) • Amounts of each white blood cell type in blood • Note: don’t just look at the percentages. Look at the absolute values too, particularly if the WBC is not exactly normal. You can get thrown off if you just look at the percentages. For example, if your patient has a WBC of 1, and the % neutrophils is 50, you might think the neutrophil count is normal (but it isn’t! That would be an absolute neutrophil count of .5, which is definitely low). • Normal ranges: ! Neutrophils Lympocytes Monocytes Eosinophils Basophils

Percentage of WBC 45-75 20-50 1-8 0-6 0-1

Absolute (x 109/L) 2-8 1-4 0.1-0.8 0-0.5 0-0.3

Platelet count (Plt) • Total number of platelets in blood • Normal range = 150-450 x 109/L • Causes of a low platelet count are numerous and include splenomegaly, idiopathic thrombocytopenic purpura, disseminated intravascular coagulation, and bone marrow failure. Causes of a high platelet count are also numerous, and include reactive thrombocytosis (as seen in iron-deficiency anemia) and essential thrombocythemia. MPV (mean platelet volum e) • Average size of platelets • Normal range depends on the platelet count! (Normally, if the platelet count falls, the body compensates a little by trying to make bigger platelets.) • Not used all that often.

www.pathologystudent.com

The Complete (but not obsessive) Hematopathology Guide page 8

The Blood Smear There are three main things you need to look at when you’re faced with a blood smear: the red cells, the white cells, and the platelets. It helps if you have a plan that you follow every time, kind of like radiologists do when they look at imaging studies. That way you’re not tempted to just start looking at the exciting stuff (like weird looking neutrophils, for example) and forget about all the other stuff. So here’s your plan.

Look at the red blood cells 1. Estimate number. • Just eyeball it; make sure there aren’t a lot of “holes” between the cells). 2. Look for variation in size (anisocytosis). • Oval macrocytes (B12/folate deficiency) • Microcytes (iron deficiency anemia, thalassemia) • The size range can often help you narrow down which type of anemia is present (for example, in iron-deficiency anemia, there is usually a big range of sizes) 3. Look for variation in shape (poikilocytosis). • Schistocytes (microangiopathic hemolytic anemia) • Spherocytes (hemolytic anemia, hereditary spherocytosis) • Teardrop cells or dacryocytes (myelofibrosis or myelophthisic processes) • Target cells or codocytes (hemoglobinopathies, thalassemias, liver disease) • Sickle cells (sickle cell anemia) • Echinocytes and acanthocytes (liver disease) 4. Estim ate the average am ount of hem oglobin in each cell (chromasia). • Normochromic (zone of central pallor comprises ! 1/3 of the cell diameter) • Hypochromic (zone of central pallor comprises >1/3 of the cell diameter) 5. Estim ate num ber of reticulocytes (look for polychrom atophilic cells). • Normal: one or two polychromatophilic cells per field. • The lower the Hgb, the higher the reticulocyte count should be. 6. Look for anything else weird. • Nucleated red blood cells • Inclusions (Howell-Jolly bodies, Pappenheimer bodies, bugs)

www.pathologystudent.com

When looking at blood smears, it helps to follow the same plan every time.

Things to make you look smart Q. What’s the difference between a polychromatophilic cell and a reticulocyte? A. “Polychromatophilic cell” and “reticulocyte” are two names for the same thing – immature red cells that still contain a little ribosomal RNA. When you do a normal Wright-Giemsa stain, the cells look big and slightly basophilic, and they are called “polychromatophilic cells.” When you do a supravital stain on a blood smear, the RNA stains blue, and the cells are called “reticulocytes.” This is one of those fine points that, when casually mentioned on rounds your third or fourth year, will make you look crazy smart.

The Complete (but not obsessive) Hematopathology Guide page 9

Look at the platelets 1. Estimate number (you should see between 7 and 21 platelets per high power field). 2. Check morphology (size, granulation).

Look at the white blood cells 1. Estimate number (you should see no more than a few white cells per high power field). 2. Check morphology (make sure the neutrophils and lymphocytes look normal, and that there aren’t any FLCs (Funny-Looking Cells, like blasts, for example, or circulating carcinoma cells). 3. Do a differential count (you should count at least 200, and preferably 500, cells). Bone marrow trephine biopsy section

The Bone Marrow Biopsy When a malignant hematopoietic disorder is suspected, a bone marrow biopsy is usually performed. The patient lies on his or her stomach, and following a little lidocaine to the skin and periosteal bone, a core sample of bone is obtained, followed by an aspiration sample of liquid marrow. The core biopsy (also called a trephine biopsy) is evaluated for cellularity (your marrow becomes more fatty as you age, just like every other body part), for the myeloid-to-erythroid ratio (which should be around 2:1), and for anything else funny looking (like lymphoid aggregates or metastatic tumor). The aspirate sample is evaluated like a blood smear; a differential count is performed (usually you count a lot more cells, like, say, 1000) and the cells are checked to make sure they look normal.

Bone marrow aspirate smear, low (top) and high (bottom) power

www.pathologystudent.com

The Complete (but not obsessive) Hematopathology Guide page 10

Special Studies Depending on what disorder you’re looking for, you may wish to order one or more special studies, which are usually performed on the bone marrow aspirate specimen. There are lots of possibilities, but here are the most common ones.

Marker

Expressed by:

CD2

all T cells

CD3

Immunophenotyping Immunophenotyping, or flow cytometry, is done on a big machine (called, not surprisingly, a flow cytometer). It’s done to look for “markers” on the surface of cells – little molecules that give clues as to what type of cell you’re dealing with. These are given CD number designations. Over on the right are some common markers.

all peripheral (postthymic, mature) T cells

CD5

all T cells

CD10

developing B cells

CD19

developing and mature B cells

CD20

developing and mature B cells

CD45

all leukocytes

CD13

granulocytes and monocytes

CD14

monocytes

CD15

granulocytes

CD33

myeloid precursors and monocytes

CD41

megakaryocytes and platelets

CD61

megakaryocytes and platelets

Cytogenetics Cytogenetic assays are performed by taking a sample of cells, growing them up in culture, and photographing them right at that point in metaphase when all the chromosomes are all lined up on the mitotic spindle so you can see them. Then you cut out the little chromosomes (on the computer), pair them up (you have to have a lot of training to do this!), and line them up in what’s called a karyotype. This is cool because it gives you a good overall look at all of the chromosomes. If there are changes like deletions or translocations, you can often see those changes in a karyotype. On the right are some common chromosomal abnormalities. The nice thing about cytogenetics is that you don’t really need to know what you’re looking for, because you’re looking at all the chromosomes (this is a great benefit in following patients with hematologic malignancies, because sometimes they develop chromosomal abnormalities that you weren’t even expecting). On the other hand, it’s not the most sensitive test in the world. So if you don’t happen to get a malignant cell in your little growing group of culture cells, then you won’t get to look at the chromosomes in the malignant population. That’s where the next set of tests comes in. Molecular studies The whole point of molecular studies is to look for a specific DNA sequence, like the particular sequence of DNA encompassing the bcr-abl translocation in CML. These tests are cool because they are generally really sensitive. Polymerase chain reaction (or PCR), in particular, is super sensitive. You only need one copy of a particular DNA sequence in your sample – PCR will amplify it over and over and over so you have a bazillion copies and you can easily detect it! This is of great benefit in certain settings, like when you’re following someone to make sure that all the disease is gone. If you can’t even detect the disease using PCR, well, then it’s really, really gone.

www.pathologystudent.com

Disorder

Abnormality

CML CLL AML-M2 AML-M3 AML-M4 AML-M4 and M5 T-cell ALL B-cell precursor ALL B-cell ALL

t(9;22) trisomy 12 t(8.21) t(15;17) inv(16) 11q23 t(11;14) t(9;22) t(8;14)

The Complete (but not obsessive) Hematopathology Guide page 11

Two. Anemia Anemia (from an-, without, and -emia, blood) is a reduction below normal in hemoglobin or red blood cell number. There are lots of different kinds of anemia, but before we leap into that discussion, lets take a look at some ways of thinking about anemia. It’s always a good idea to have an underlying understanding, or plan, when you’re approaching a sh*tload of diseases. Who can memorize 42 separate things? Well, okay, you can, or you wouldn’t be here. But it makes it easier if you can group those 42 things together into, say, 3 or 4 bigger categories.

Causes One way to group the anemias is by their cause. If you think about it, there are basically only three ways you can get anemic. All the anemias we’ll talk about fit into these three mechanisms: 1. Lose blood (for example, as a result of major trauma) 2. Destroy too much blood • Extracorpuscular hemolytic disease (e.g., autoimmune disorders, trauma to red blood cells) • Intracorpuscular hemolytic disease (e.g., hereditary membrane or globin problems) 3. Make too little blood • Bad diet (not enough iron, B12, or folate) • Decreased number of erythroblasts (as in aplastic anemia) • Bone marrow full of other stuff besides hematopoietic precursors (e.g., tumor) • Chronic disease (e.g., renal disease, inflammatory diseases)

www.pathologystudent.com

The Complete (but not obsessive) Hematopathology Guide page 12

Morphologic groups Another way to think about the anemias is in terms of the way they look on a blood smear. Which actually makes some sense, because when a patient comes in, he or she doesn’t say, “I have an anemia due to lack of folate” – what happens is that you suspect anemia, and then look at their blood smear to try to figure it out. It doesn’t really matter which little grouping scheme you choose, just pick one that makes sense to you. In this study guide, the anemias are loosely organized by the morphologic group scheme. 1. Anemias with abnormally-sized red cells • Red cells too small (microcytic) (iron-deficiency anemia, thalassemia) • Red cells too big (macrocytic) (megaloblastic anemia) 2. Anem ias with abnorm ally-shaped red cells • Round red cells (spherocytes) (hereditary spherocytosis, autoimmune hemolytic anemia) • Pointy red cells (sickle cell anemia, G6PD deficiency, microangiopathic hemolytic anemia) • Target-shaped red cells (hemoglobinopathies, thalassemias) 3. Anemias with normal-looking red cells • Increased reticulocytes (massive hemorrhage that occurred over 3 days ago) • No increase in reticulocytes (recent massive hemorrhage, anemia of chronic disease, aplastic anemia)

Signs and symptoms One final note before we get on to the specific anemias. Patients with anemia can present in different ways, depending on what kind of anemia they have and how severe it is. The general signs and symptoms of anemia relate to the underlying lack of oxygen-carrying capacity: fatigue, weakness, dizziness, tachycardia, pallor of skin and mucous membranes. In addition to the general symptoms of anemia, some specific findings may be present. If the anemia is hemolytic, the patient may be jaundiced. Patients with iron-deficiency anemia may show spoon-shaped nails (koilonychia), a smooth tongue, or pica (a craving to eat dirt and other nonfood items). And patients with megaloblastic anemia may develop a big, beefy tongue, or loss of vibratory sensation or proprioception. It’s important to remember, however, that if an anemia is fairly mild, symptoms may not be present at all. Also, if the anemia is chronic and slowly-progressive, the cardiovascular system adjusts to the new diminished level of oxygen, and symptoms will only appear when the anemia becomes quite severe.

www.pathologystudent.com

The Complete (but not obsessive) Hematopathology Guide page 13

Iron-Deficiency Anemia Iron-deficiency anemia is a common type of anemia. One of the most common causes is blood (and hence iron) loss – so it’s important to work up any patient to find a source of blood loss, if it exists. Occasionally, colon cancer presents with iron-deficiency anemia due to chronic loss of small amounts of blood in the stool.

Iron Facts Absorption Absorption of iron occurs in duodenum/proximal jejunum. In the mucosal cell, iron is bound to either ferritin (for storage) or transferrin (for circulation). Circulation Iron is bound to transferrin, which carries iron to red blood cell precursors in bone marrow, and to other organs. Distribution Most of the iron in the body is in hemoglobin; a smaller percentage is in storage forms (ferritin and hemosiderin), and a very very small amount is bound to transferrin. Metabolism Most of the circulating iron is taken up by red cell precursors and incorporated into heme (which is then combined with globin chains to make hemoglobin). The rest of the iron is stored in macrophages in the marrow, spleen, and liver. Storage There are a couple ways iron is stored: in ferritin (a labile iron storage form – quick in, quick out) and in hemosiderin (a stable storage form that contains ferritin and cell debris).

www.pathologystudent.com

The Complete (but not obsessive) Hematopathology Guide page 14

Causes of Iron Deficiency Decreased iron intake Dietary deficiency is rarely the sole cause of iron deficiency anemia (unless you are eating a really crappy diet). More likely is decreased absorption, for example, in patients who have had gastric surgery. Increased iron loss Here’s the scary potential cause of iron deficiency: GI bleeding (e.g., from gastric ulcer, colon cancer). This is why you never just treat an iron-deficient person with iron without figuring out the cause of the iron deficiency! You might miss something really important that way. Other causes of bleeding that are readily identifiable and usually less scary include excessive menstrual flow (menorrhagia) and acute blood loss (e.g., massive trauma, childbirth). Increased iron requirement Women who are pregnant need more iron; if their intake doesn’t increase accordingly, a deficiency can arise. It all boils down to this: 1. IDA in premenopausal women: First thing to consider is menorrhagia. 2. IDA in men and postmenopausal women: First thing to consider is GI blood loss.

Bottom line: IDA in men and postmenopausal women: rule out GI blood loss.

Clinical Features

Things to make you look smart

Sym ptom s Typical symptoms of iron-deficiency anemia, like any type of anemia, are related to the lack of ability to carry oxygen around! So patients have fatigue, palpitations, dizziness, and breathlessness. Note, though, that if the anemia is mild (say, with a hemoglobin over 8), or if it’s long-standing (say, from a slow GI bleed), the patient might have few or no symptoms at all!

Q. Why does the patient's hemoglobin appear normal immediately after a big blood loss?

Signs Patients with anemia in general (whatever the cause) may show pale skin and mucous membranes. Patients with anemia due to iron deficiency may have some unique symptoms, such as thinned, flattened, spoon-shaped nails, or a smooth, shiny tongue. A really strange symptom of iron deficiency that occurs in some patients is pica. Hippocrates described pica as "a craving to eat the earth" associated with "corruption of the blood." Good description. For whatever reason, some patients with severe iron deficiency get cravings to eat weird stuff, like dirt, ice, cardboard, and Windex. Of course, there are other causes of pica too, some of them psychiatric in nature.

www.pathologystudent.com

A. Because the patient has lost not only red cells, but plasma. So the blood that's left in the body has the same concentration of hemoglobin as it did before the blood loss. After a while, plasma volume is restored (either artificially or by the body itself), and the hemoglobin (now diluted) is decreased.

The Complete (but not obsessive) Hematopathology Guide page 15

Morphology Blood • Hypochromic, microcytic anemia • Increased anisocytosis (some little cells, some bigger cells) • Increased poikilocytosis (elliptocytes are present, for some reason) • Decreased reticulocyte number (because there’s not enough iron around!) • Platelet count usually increased for some reason

Iron-deficiency anemia is a

microcytic, hypochromic anemia with

oval macrocytes increased anisocytosis and poikilocytosis.

Bone marrow • Erythroid hypoplasia • Dyserythropoiesis (funny looking red precursors)

Iron studies • • •

" serum iron # TIBC (total iron binding capacity) " ferritin (But remember: ferritin is an acute phase reactant, which means it goes up in certain settings, like inflammation. So a normal ferritin (or even an increased ferritin) doesn’t rule out iron deficiency!)

Treatment Figure out why patient is iron deficient (don't just treat the anemia, or you might miss something really important). Then give iron (orally).

www.pathologystudent.com

Iron-deficiency anemia

The Complete (but not obsessive) Hematopathology Guide page 16

Megaloblastic Anemia Patients with megaloblastic anemia are unable to make DNA at a normal rate, leading to big, weird-looking cells with immature nuclei but mature cytoplasm. The most common cause is B12 and/or folate deficiency.

Pathogenesis Retarded DNA synthesis (causing cells to divide more slowly), but unimpaired RNA synthesis (allowing cytoplasm to mature at normal speed) leads to big cells with immature nuclei but mature cytoplasm (nuclear/cytoplasmic asynchrony). Vitam in B 12 and/or folate deficiency is most common cause of retarded DNA synthesis. You need both B12 and folate to make DNA: Methyl FH4

B 12 FH4

Methylene FH4 dUMP

www.pathologystudent.com

FH2 dTMP

DNA

The Complete (but not obsessive) Hematopathology Guide page 17

Vitamin B 12 Sources B12 is found in meat, dairy products, and fortified foods, like breakfast cereal, but not in plants! How does B 12 get to red cells? Ingested B12 binds to intrinsic factor (secreted by gastric parietal cells). B12 /IF is absorbed in the distal ileum. B12 is transported by transcobalamin to organs and erythroblasts. W hat else do you need B 12 for? You need B12 to convert B12 is also necessary for conversion of homocysteine to methionine. You need methionine for homocysteine into methionine, myelin maintenance (patients with untreated B12 deficiency eventually get an irreversible and you need methionine for demyelinating disease of the spinal cord called “subacute combined degeneration”). So…even if myelin maintenance. you know a patient has a folate deficiency, always check for a concurrent B12 deficiency! Causes of B 12 deficiency 1. Diet. This is a pretty rare cause. If you stopped eating B12 completely, it would take a few years to become anemic. 2. Pernicious anemia. Patients have autoantibodies to their parietal cells (as these are destroyed, less IF is produced). Once dreaded and lethal (“pernicious”), now easily treated with B12 injections. To see how well a patient absorbs B12, you can do a Shilling test. 3. Pancreatic damage. Lack of pancreatic enzymes means you can’t liberate B12 in the stomach, which means that B12 can’t bind to IF. 4. Ileal disease/resection. Without an ileum, B12-IF can’t be absorbed. 5. Bugs in the small intestine. Tapeworms or bacterial overgrowth can compete for B12.

Things to make you look smart Q. How do you do a Shilling test? A. 1. Give “flushing” dose of intramuscular B12. 2. Give small oral dose of radioactive B12. • Healthy patients will excrete radioactive B12 in urine. • Patients who can’t absorb B12 via gut will not excrete radioactive B12 in urine. 3. If urine has low radioactivity, give another oral dose of radioactive B12, this time with intrinsic factor. • If patient now excretes radioactive B12, patient lacks intrinsic factor. • If patient still doesn’t excrete radioactive B12, the defect is not in intrinsic factor (something else is probably wrong with absorption).

www.pathologystudent.com

The Complete (but not obsessive) Hematopathology Guide page 18

Folate Sources There are lots of dietary sources of folate – like green leafy vegetables (“folic” means leafy), legumes, yeast, organ meats, fruit, eggs, and fortified products like breakfast cereals. How does folate get to red cells? Folate is absorbed mostly in the jejunum. It’s converted to methyl-FH4 during absorption, and then transported freely (mostly) to liver, red blood cells.

B12 stores are big,

but folate stores are small.

Causes of folate deficiency 1. Diet. Folate stores are relatively small! If you stopped eating folate completely, it would only take a few months to become anemic. 2. Alcohol abuse. Probably due to malnutrition, poor absorption of folate, and/or inhibition of folate metabolism. 3. Jejunal disease. For example, sprue, inflammatory bowel disease, jejunal resection. macrocytic 4. Drugs. Especially chemotherapeutic drugs, many of which are folate antagonists.

Megaloblastic anemia is a anemia with oval macrocytes and hypersegmented neutrophils.

Morphology Blood • Macrocytic anemia (MCV >100) • Oval macrocytes. • Hypersegmented neutrophils. Bone marrow • Megaloblastic erythroblasts (BIG cells with big, immature nucleus but maturing cytoplasm). • Megaloblastic neutrophils and precursors. • Giant metamyelocytes. • Hypersegmented neutrophils.

www.pathologystudent.com

Megaloblastic anemia

The Complete (but not obsessive) Hematopathology Guide page 19

Hemolytic Anemias Causes Inherited • Defects in red blood cell membrane (e.g., hereditary spherocytosis) • Enzyme deficiencies (e.g., glucose-6-phosphate dehydrogenase deficiency) • Defects in globin structure/synthesis (e.g., hemoglobinopathies)

In hemolytic anemias, there is an

Acquired • Autoimmune hemolytic anemia • Microangiopathic hemolytic anemia

increased rate of red cell destruction, and (hopefully) an increased rate of red cell production

Clinical features Chronic hemolytic anemias These are usually congenital, and are often well-compensated, with few symptoms. When something happens to disturb the fragile equilibrium between red cell destruction and bone marrow compensation, though (for example, infection with parvovirus B19), the already maxed-out bone marrow cannot compensate, and the patient goes into a hemolytic crisis. Symptoms of such crises include jaundice (from an increase in unconjugated bilirubin), splenomegaly, and gallstones. Acute hem olytic anem ias These are usually acquired, and present comparatively suddenly with symptoms like backache, abdominal pain, headache, malaise, fever, pallor, jaundice, and tachycardia.

Laboratory findings Signs of excessive red cell destruction • Hemoglobinemia/hemoglobinuria (when hemolysis is intravascular, or so brisk extravascularly that macrophages cannot keep up). • # serum unconjugated bilirubin (unless liver can keep up with excretion, in which case it will be conjugated!). • # lactate dehydrogenase (LDH) (a red cell enzyme). • " haptoglobin (a protein that binds free Hgb).

Reticulocytes

Signs of accelerated erythropoiesis: reticulocytosis Reticulocyte count = percentage of red blood cells that are reticulocytes (normal=1-3%). The lower the hemoglobin, the higher the reticulocyte count should be!

www.pathologystudent.com

The Complete (but not obsessive) Hematopathology Guide page 20

Morphology • • •



Normochromic, normocytic (usually) anemia. Spherocytes (found in almost every hemolytic process). Other more specialized poikilocytes: • Target cells (thalassemias and hemoglobinopathies) • Elliptocytes (hereditary elliptocytosis) • Sickle cells (sickle cell anemia) • Schistocytes (microangiopathic hemolytic anemia) Signs of increased erythropoiesis: • Polychromatophilia (or, if stained with supravital stain, reticulocytosis) • Basophilic stippling (RNA remnants not yet removed from cell) • Nucleated red blood cells (bone marrow hurrying to get red cells out as quickly as possible)

Important lab test Direct antiglobulin test (DAT) Also called the Coomb's test. Mix patient's red cells with anti-human globulin (an antibody against human immunoglobulins). If the patient’s red cells are coated with antibodies (as they are in some immune processes, see later), the anti-human globulin will attach to those antibodies, bridging the red cells and making them clump together. So, a positive result (red cell clumping) means the patient's red cells are coated with antibodies, and the hemolysis is probably immune-related.

How to diagnose a hemolytic anemia First, is there hem olysis? Look for signs of increased red cell destruction (like increased bilirubin) and signs of increased erythropoiesis (if hemolysis has been around long enough). Second, what's causing the hem olysis? Using the history, DAT, and blood smear, you can categorize patients into five groups: 1. Patients with known exposure to infectious or chemical agents. 2. Patients with positive DAT (diagnosis: immune-related hemolytic anemia). 3. Patients with negative DAT but lots of spherocytes (probable diagnosis: hereditary spherocytosis). 4. Patients with negative DAT and other specific, morphologic abnormalities (e.g., sickle cells). 5. Patients with negative DAT and no specific morphologic abnormalities (do Hgb electrophoresis).

www.pathologystudent.com

The Complete (but not obsessive) Hematopathology Guide page 21

Hereditary Spherocytosis HS is a relatively common hemolytic anemia (1 in 5000 people of northern European descent) with a variable age of onset and severity. Patients often (but not always) have a classic triad of mild anemia, intermittent jaundice and splenomegaly. Anemia is exaggerated during crises, which are frequently precipitated by infection with parvovirus B19 (fifth disease).

Pathogenesis The basic defect is in a protein called spectrin, which helps anchor the red cell membrane to the cytoskeleton. Patients with HS have defective spectrin, which leads to altered membrane properties (loss of surface area, altered membrane lipids/proteins) and the formation of spherocytes (which get eaten up by macrophages in the spleen).

Hereditary spherocytosis: Lots of spherocytes due to a spectrin defect.

Morphology • • •

Mild normochromic, normocytic anemia. Numerous spherocytes. Evidence of accelerated hematopoiesis (polychromatophilia, normoblasts).

Treatment Splenectomy cuts way down on the symptoms (because that’s where the red cells get destroyed). If splenectomy isn’t possible, the patient may need red cell transfusions during crises.

Other hemolytic anemias due to inherited membrane abnormalities Hereditary elliptocytosis Patients with HE have a spectrin abnormality too, but the mutation is different than that in HS. The clinical features are similar to those in HS, but the blood smear shows tons of elliptocytes.

Hereditary spherocytosis

Hereditary pyropoikilocytosis In this disorder, there is a different problem with spectrin: it won't stick to itself, and therefore gets degraded too rapidly. The clinical picture is pretty similar to HS and HE, but the morphology is different: the red cells take on all kinds of bizarre shapes. They look like red cells that have been exposed to heat – thus the name "pyro".

www.pathologystudent.com

The Complete (but not obsessive) Hematopathology Guide page 22

Glucose-6-Phosphate Dehydrogenase Deficiency Glucose-6-phosphate dehydrogenase deficiency is a type of anemia that only appears after exposure to certain oxidizing agents, like fava beans, or certain medications. Following exposure, the red cells appear to have “bites” in them, from removal of inclusions called Heinz bodies.

Pathogenesis G6PD is a pretty important enzyme. It catalyzes the initial step in the pentose phosphate pathway of glycolysis. Plus, you need G6PD to reduce NADP to NADPH, which in turn keeps glutathione in the reduced state (reduced glutathione detoxifies hydrogen peroxide and other organic peroxides). You normally make free radicals during routine cell life. When you get exposed to certain oxidizing agents, though, you make a ton more. When there isn’t enough G6PD around, these free radicals attack sulfhydryl groups and break disulfide bonds in the cell. Heme is liberated from globin, and globin is denatured, making a little round inclusion called a Heinz body which sticks to the red cell membrane and compromises membrane plasticity. The red cells are detained in their passage through the liver and spleen, where macrophages remove Heinz bodies, leaving a little “bite” in the red cell.

Without G6PD, free radicals accumulate and Heinz bodies form.

The gene for G6PD is on the X chromosome. Therefore, males with G6PD deficiency usually have full disease expression, and heterozygous females are clinically normal. G6PD deficiency is more common in certain populations (10% of black men in US have the gene). The highest incidence is in populations in which malaria has been endemic. It may confer a protective advantage against malaria because G6PD-deficient red cells lack the ribose derivatives bugs need to grow.

Morphology Without exposure to offending agents, most patients have no anemia. After exposure, though, patients get an acute hemolytic episode, with cell fragments, microspherocytes, and bite cells (caused by recent pitting of Heinz bodies). Supravital staining reveals Heinz bodies (these decrease in number as Hgb bottoms out, because younger cells have greater G6PD activity).

Bite cell

Treatment In most affected individuals, severe hemolysis occurs only after exposure to oxidizing agents (such as antimalarial drugs, sulfonamides, H2O2, aspirin, and fava beans) or infection (by unknown mechanisms). So it’s important to avoid exposure to known oxidants. Usually the hemolysis is selflimiting, with spontaneous resolution in a week or so, and no specific treatment is required.

www.pathologystudent.com

The Complete (but not obsessive) Hematopathology Guide page 23

Sickle-Cell Anemia Sickle-cell anemia is one of a group of inherited disorders called hemoglobinopathies in which structurally abnormal hemoglobin is made. Some of these abnormal hemoglobins function like normal hemoglobin, but some have abnormalities that lead to hemolysis. Hgb S is the most common of these; Hgb C is another fairly common one. Patients with two Hgb S genes are said to have sickle-cell anemia, a disease characterized by hemolysis and microvascular occlusion.

Sickle cell anemia is a

Pathogenesis Most hemoglobinopathies are caused by a point mutation in a $ chain gene. Weird how just one little point mutation can cause such a catastrophic disease. In the case of sickle-cell disease, the mutation leads to a substitution of valine for glutamate in position 6 in the $ chain molecule, which results in an abnormal hemoglobin, Hgb S, with abnormal physical and chemical properties. The big problem with Hgb S occurs during deoxygenation. As the Hgb S molecules give up oxygen, they aggregate and polymerize, distorting the red cell into a sickle shape. After many such deoxygenation episodes, the red cell becomes permanently sickle-shaped.

qualitative abnormality of hemoglobin

This is not a great fate for the red cell because sickles are not as deformable as normal biconcave disk-shaped red cells. Normal red cells flow easily through small vessels, but sickles are much more likely to get stuck and pile up on each other, occluding vessels and causing ischemic tissue damage. In addition, sickles are more fragile than regular red cells, so there’s an element of chronic hemolysis going on too.

Clinical Features Sickle cell disease occurs predominantly in blacks. Eight percent of blacks in US are heterozygous (heterozygosity is called sickle cell trait and rarely leads to significant clinical or hematologic manifestations); one black child in 600 is homozygous (homozygosity is called sickle cell disease and the severity varies quite a bit from patient to patient, for unknown reasons). Patients have chronic hemolysis (the typical hemoglobin ranges from 6-10). Vaso-occlusive disease usually occurs as an acute crisis, precipitated by infection, hypoxia, or other unidentified causes. In children, pain in the hands or feet is often the first symptom. In all ages, bone, lung, and abdominal pain is common.

www.pathologystudent.com

The Complete (but not obsessive) Hematopathology Guide page 24

The spleen may undergo massive enlargement (due to red cell sequestration) in early childhood. By early adulthood, however, the spleen is reduced to a small, fibrotic remnant (due to recurrent hemorrhage and fibrosis); this is called “autosplenectomy.” Because the spleen is so critical in fighting off encapsulated organisms, it’s important to vaccinate patients with sickle-cell disease against bugs like Streptococcus pneumoniae and Haemophilus influenzae.

Morphology In the blood, particularly during crises, sickle cells are present. After autosplenectomy occurs, there is what’s called a "post-splenectomy blood picture": nucleated red blood cells, targets, Howell-Jolly bodies, Pappenheimer bodies, and a slightly increased platelet count (the platelets love to hang out in the spleen, so when you take away their little home, they have no choice but to hang out in the blood).

Treatment Treatment is tailored to each patient. It’s important to prevent triggers (which are different in different patients, but include things like infection, stress, fever, dehydration, and hypoxemia). Extra precautions must be taken to prevent infections due to encapsulated bugs (vaccinate, give prophylactic antibiotics). Blood transfusions might be necessary during severe crises (maybe also regularly, in certain patients). As an absolute last resort, bone marrow transplantation can be used (but it has a relatively high mortality, so it’s not for everyone).

www.pathologystudent.com

Sickle-cell anemia

The Complete (but not obsessive) Hematopathology Guide page 25

Thalassemia The thalassemias are characterized by a quantitative decrease in one of the hemoglobin chains. In %-thalassemia, there is a decreased amount of % chain. In $-thalassemia, there is a decreased amount of $ chain. You end up with a two-fold problem: 1. Decreased hemoglobin production (because of the decrease in globin chains) 2. Excess unpaired % chains (in $ thalassemia) or $, &, and ' chains (in % thalassemia), which form tetramers and lead to premature red cell destruction. Let’s look at this in a little more depth.

Thalassemia is a

quantitative abnormality of hemoglobin

Pathogenesis Normal hemoglobin composition Before 6 months of age, most of a person’s circulating hemoglobin is fetal hemoglobin (HgbF), which has two % chains and two & chains (%2&2). Around 6 months of age, hemoglobin composition starts switching to the adult pattern, which is a composite of three types: 96% is HgbA (%2$2), 3% HgbA2 (%2'2), and 1% HgbF (%2&2). The genes for the % and $ chains differ in number; there are four %-chain genes and two $-chain genes. Disease severity depends on how m any genes are defective (or absent). In $-thalassemia, the defect is in transcription, translation, or processing of mRNA. The defective $ gene is designated differently depending on the severity of the defect. A normal $ chain gene that produces normal amounts of $ chains is designated a $ gene. A gene that produces no $ chains is designated as $0, and a gene that produces some $ chains (but less than a normal amount) is designated as $+ . So, depending on what kinds of genes you have, you can have anything from asymptomatic $ thalassemia to really severe $ thalassemia. (Notice that $0/$ appears twice in the chart below! That’s because there is a variable clinical picture; not everyone with the same genetic defect has exactly the same symptoms.)

www.pathologystudent.com

The Complete (but not obsessive) Hematopathology Guide page 26

if genotype is:

disease is:

$0/$0 or $0/$+ $0/$ or $+/$+ $0/$ or $+/$

$-thalassemia major (severe) $-thalassemia intermedia (less severe) $-thalassemia minor (mild or asymptomatic)

In %-thalassemia, the "defect" is really gene absence, so there are no different symbols or designations for different kinds of % genes. You just put a little dash to indicate a missing gene. Fortunately, there are four % genes – so you can delete one or even two of them and still have a normal life. if genotype is:

disease is:

--/%% or -%/-% --/-% --/--

%-thalassemia trait (asymptomatic) HbH disease (severe) Hydrops fetalis (fatal in utero)

So to summarize: %-thalassemia In %-thalassemia, the problem is deletion of one or more gene. The anemia is caused by two things: 1. Insufficient % chains (leading to insufficient Hgb A) 2. Excess unpaired $, &, and ' chains, which form tetramers. Newborns have higher % of HgbF, so they make &4 tetramers (called “Hb Barts”), whereas adults have mostly HgbA, so they make $4 tetramers (called “HbH”). Macrophages in the spleen gobble up these tetramer-laden red cells. Yummy. $-thalassemia In $-thalassemia, the problem is in transcription, translation, or processing of mRNA. The anemia is caused by two things: 1. Insufficient $ chains (leading to insufficient Hgb A) 2. Excess unpaired % chains (leading to premature red cell destruction because splenic macrophages see these as yummy)

www.pathologystudent.com

Things to make you look smart Q. Both IDA and thalassemia are hypochromic and microcytic. So how can you tell them apart morphologically? A. There are two morphologic clues: 1. In IDA, there is usually a fair amount of anisocytosis (which, if you recall from our discussion of lab tests in the beginning, is reflected in the RDW). Most cases of thalassemia (except for the really severe ones) show a very minimal amount of anisocytosis. In fact, the cells are virtually all the same exact size! Weird. So: IDA has an increased RDW, and thalassemia has a normal (or decreased) RDW. 2. In IDA, the RBC is low (which makes sense, because there’s not enough iron around to make red cells properly). Most cases of thalassemia, however, have an increased RBC, for some unknown reason! Weird. So: IDA has a decreased RBC, and thalassemia has an increased RBC. Of course, these are just morphologic clues. To really make an official diagnosis, you’d need to do iron studies (to look for iron deficiency) and hemoglobin electrophoresis (to look at the types and amounts of hemoglobin present).

The Complete (but not obsessive) Hematopathology Guide page 27

Morphology %-thalassemia Patients with silent carrier state and %-thalassemia trait have no anemia. Patients with HbH disease have moderate to severe anemia with a range of changes similar to those described for $thalassemia, below. Supravital staining reveals HbH inclusions. $-thalassemia Patients with $-thalassemia minor have a mild microcytic, hypochromic anemia. There is usually some basophilic stippling (blue dots in the red cells), and target cells.

Thalassemias are

microcytic and hypochromic and

often have decreased anisocytosis

Patients with $-thalassemia intermedia and $-thalassemia major have a severe anemia (Hgb=3-6) with a very abnormal-looking smear. The red cells show marked anisocytosis and poikilocytosis, and there are usually a bunch of nucleated red blood cells (as the marrow is trying hard to pump out any red cells it makes as soon as it makes them).

Clinical findings %-thalassemia %-thalassemia is more common in Asians and Blacks. The silent carrier state and %-thalassemia trait produce no symptoms. HbH disease produces a moderately severe anemia, and hydrops fetalis is fatal in utero. $-thalassemia $-thalassemia is more common in Mediterraneans, Blacks and Southeast Asians. Patients with $thalassemia minor are almost always asymptomatic, with mild or no anemia. Patients with $thalassemia major have a very severe very severe, starting at 6-9 months of age. Without therapy, children suffer growth retardation and die at a young age from profound anemia. Treatment involves repeated transfusions (with iron chelation to avoid iron overload). Unless bone marrow transplant is performed, many patients die in their 20s.

Moderate thalassemia

Treatment Patients with mild thalassemia don’t require treatment. Patients with severe anemia may need repeated red cell transfusions or even bone marrow transplantation.

www.pathologystudent.com

The Complete (but not obsessive) Hematopathology Guide page 28

Warm Autoimmune Hemolytic Anemia The autoimmune hemolytic anemias (AIHAs) are disorders in which the patient makes autoantibodies against his or her red cells, leading to early red cell destruction. There are two flavors of AIHA, warm and cold, so-named for the temperature at which these autoantibodies like to react. This sounds arcane, but it really does matter, as we’ll see below. If the antibody reacts at a warm temperature, it can glom onto the red cell anywhere in the body, whereas if it reacts at cool temperatures, it will bind to the red cell only in cool parts of the body and then fall off in warm parts. This changes the way the disease presents. Let’s start with warm AIHA.

Pathogenesis Most cases of warm AIHA are primary, or idiopathic (meaning: we have no idea what is causing it). A smaller number of cases are secondary to things like leukemia or lymphoma, other malignancies, autoimmune disorders, infections, or drugs (like methyl-dopa or penicillin). For whatever reason, the patient makes IgG Ab that bind to his or her red cells. The binding occurs best at 37° C, which just happens to be normal body temperature. Macrophages in spleen think these IgG-coated (“opsonized”) red cells are yummy, and they nibble out bits of red cell membrane, which makes the cells round up into spherocytes. Eventually, the macrophages eat up the spherocytes entirely.

Clinical Features Warm AIHA hits patients of any age or race, and either sex. Patients have a variably severe anemia, and often there is some degree of splenomegaly (because of all the nom-noms).

WAIHA:

IgG spleen spherocytes Things to make you look smart Q. Why is this coating process called “opsonization?” A. Opsonization comes from the Greek word ὄψον (opson), which describes the yummiest part of a meal (like the relish that is eaten alongside the more boring stapletype food). This is a great word root to use in this context, because things that are opsonized (coated with antibody or complement) look particularly delicious to hungry macrophages, who go after such cells with gastronomic gusto. Mmmmmm…antibodylicious.

Morphology The blood smear shows prominent spherocytosis and other signs of hemolysis (polychromasia, normoblasts).

Diagnosis To make a diagnosis, you need to order a direct antiglobulin (Coombs') test (DAT), which detects IgG and/or complement bound to red cells. Patients with warm AIHA are positive for IgG.

Treatment Treat underlying cause, if there is one. Steroids can be useful, and if all else fails, splenectomy might be necessary.

www.pathologystudent.com

WAIHA

The Complete (but not obsessive) Hematopathology Guide page 29

Cold Autoimmune Hemolytic Anemia Pathogenesis Just as in WAIHA, there are a bunch of possible etiologies in CAIHA. About half of the cases are primary; the rest are secondary to infections (like mycoplasma pneumoniae or infectious mononucleosis) or lymphoproliferative disorders (like lymphoma). Patients make anti-red-cell antibodies that are IgM in nature (instead of IgG). For some reason, they are usually directed against the "I" antigen on the red cell surface. These bind to the red cell best at temperatures 100). There’s an increased amount of poikilocytosis, with target cells (in chronic liver disease, there is increased cholesterol and lecithin in the red cell membrane, which leads to increased cell surface area) and acanthocytes, or "spur" cells (red cells with 5-10 long, spiky surface projections).

Clinical Three-fourths of patients with liver disease are anemic! Interesting fact: alcohol abusers with even mild liver disease can get episodes of hemolysis which resolve when alcohol is withdrawn. Makes you think.

www.pathologystudent.com

The Complete (but not obsessive) Hematopathology Guide page 36

Aplastic Anemia Aplastic anemia is a disease in which the bone marrow is “empty” – that is, there are very few, if any, hematopoietic precursors. It’s really more than just an anemia; it’s a pancytopenia (meaning that all the major cell lines – red cells, white cells, platelets – are severely decreased).

Pathogenesis The list of possible causes is long – but most of the time, no specific cause can be identified. Of the remaining cases, most are acquired (meaning you’re not born with it), and are due to things like drugs (e.g., chloramphenicol), viruses (e.g., HIV), pregnancy, or radiation. A small number of cases are familial. The most common of these familial causes of aplastic anemia is Fanconi anemia, a hereditary disease characterized by skeletal abnormalities, chromosomal instability, pancytopenia and increased risk of leukemia.

Patients with aplastic anemia have pancytopenia and an empty marrow.

Clinical features Fortunately, aplastic anemia is a rare disease. The death rate is approximately 1-13 people/1,000,000 per year. It is not age- or sex-related. The signs and symptoms are related to the pancytopenia. Patients are pale, and feel fatigued and dizzy (from the anemia); they may get recurrent infections (from the leukopenia); and they may have excess bleeding or bruising (from the thrombocytopenia).

Morphology Blood The blood is pancytopenic, meaning that the red cells, white cells, and platelets are all decreased. The anemia itself is “bland” – it’s normochromic and normocytic with minimal anisopoikilocytosis. Bone marrow The bone marrow is markedly hypocellular, or "empty". There is a lot of fat, and a few lymphocytes, but very very few red cell, white cell, and platelet precursors. The bone marrow aspirate specimen consists mostly of lymphocytes, with some mature red cells – but very few hematopoietic precursors.

www.pathologystudent.com

Aplastic anemia: bone marrow

The Complete (but not obsessive) Hematopathology Guide page 37

Differential diagnosis If all you have to go on is a pancytopenic blood smear (no bone marrow biopsy), then in addition to aplastic anemia, you’d have to consider disorders infiltrating the bone marrow (e.g., myelofibrosis, metastatic carcinoma), disorders involving the spleen (e.g., congestive splenomegaly, lymphoma) and other miscellaneous disorders (e.g., overwhelming infection, and something called “refractory anemia” which we’ll mention when we get to myelodysplastic syndromes). To get to the bottom of things, you’d really need a bone marrow biopsy.

Treatment and prognosis Obviously, if the anemia is the result of exposure to a known noxious agent, it would be a good idea to avoid further exposure. More specific treatment may include transfusion of blood components as needed (red blood cells, platelets), drug therapy to stimulate hematopoiesis (GCSF, steroids, androgens) and suppress the immune system (anti-thymocyte globulin), and if necessary, bone marrow transplant. The 5-year survival rate is around 70-90%.

www.pathologystudent.com

The Complete (but not obsessive) Hematopathology Guide page 38

Three. Benign Leukocytoses There are lots of benign reasons for an elevated white cell count (leukocytosis). There are, as you know, several different kinds of white cells, each with its own unique characteristics and functions – so there are several different types of benign leukocytosis, each with its own causes and implications. We’ll discuss these separately, with emphasis on neutrophilia and lymphocytosis.

Benign Neutrophilia Where do neutrophils live? Most neutrophils live in the bone marrow. Only about 5% live in the blood. Here’s a weird fact: half of the neutrophils in the blood are “marginated” – meaning that they are plastered up against the walls of the vessels! This is important because if you get emotionally stressed (like, say, in the emergency room, or during a particularly bad pathology exam), you can pull most of those marginated neutrophils off the walls of the vessels and into the bloodstream, effectively doubling your neutrophil count. Something to remember when you see a child in the emergency room with a high neutrophil count.

Normal neutrophil count The normal range for neutrophils is between 2 and 8 x 109/L. Like most tests with a reference range, it varies a bit by institution or laboratory – so you’d need to check your exact range. There are some physiologic variations in the neutrophil count. Hormones can change the neutrophil count (it goes up in pregnancy and menstruation and down after menopause), as can the typical medical or dental student trifecta of stress, smoking and alcohol (all of which raise the neutrophil count). There’s some diurnal variation too, with counts being higher in the evening than in the morning.

www.pathologystudent.com

The Complete (but not obsessive) Hematopathology Guide page 39

Proliferation of mature neutrophils When you have a patient with a high neutrophil count, it pays to look and see what kind of neutrophils are proliferating – mature ones (segmented neutrophils, the ones you usually see in peripheral blood), or immature ones (like bands, metamyelocytes, myelocytes, promyelocytes– cells you usually see only in the marrow). The causes of a mature neutrophilia are slightly but significantly different than the causes of an immature neutrophilia. If all you see are mature neutrophils, the main causes you should consider are infection (usually bacterial, not viral) and inflammation. Sometimes, in infections, the neutrophils will show odd changes called “toxic changes.” These include so-called toxic granulation (abundant, deep dark granules in the cytoplasm), Döhle bodies (pretty, sky-blue blobs in the cytoplasm, probably representing revved-up rough endoplasmic reticulum), and cytoplasmic vacuolization (just what the name says). You can see these changes in any type of bacterial infection, but they are especially abundant in serious infections like sepsis. Toxic granulation (L) and vacuolization (R)

Proliferation of immature neutrophils If you see immature neutrophils, then it’s a different story. There are three different kinds of immature neutrophilia: left shift, leukemoid reaction (bad term! bad term!), and leukoerythroblastotic reaction. Left shift A “left shift” just means that there are immature neutrophils in the blood. The causes of a left shift are fairly similar to the things we talked about above under mature neutrophilia, namely, infection and inflammation. Leukem oid reaction This is a bad term! Don’t use it! The reason it’s bad is because it doesn’t have a single, clear definition. The word “leukemoid” means that when you look at the blood, it looks like leukemia but it isn’t. So that could mean that there’s a very high neutrophil count, or it could mean that there’s a marked left shift, or it could mean both…see how confusing it is? You’ll hear some older physicians use the term (and you might read it on some websites of questionable repute); you’d have to figure out what the actual morphologic changes are in order to make some sense out of it. Or you could just walk away, shaking your head and smiling.

www.pathologystudent.com

Left shift

The Complete (but not obsessive) Hematopathology Guide page 40

Leukoerythroblastotic reaction Now this is not a bad term. It has a very well-established definition: it means that there are both immature neutrophil precursors and immature red cell precursors in the blood. By the way, anything other than a regular old red cell would be considered immature in the blood. Any red cell precursor with a nucleus, no matter what the stage of development, is normally seen only in the marrow. There is a different set of causes for this particular kind of immature neutrophilia, and not all of them are nice causes. Sometimes, in situations when the hemoglobin is really low (like, say, under 6), the bone marrow is so bent on getting red cells out into the periphery that it ends up letting a few nucleated red cells slip out along with all the mature red cells. The marrow is a little sloppy sometimes, and on occasion, it might let some immature neutrophils out at the same time. In that setting, a leukoerythroblastotic reaction (LEBR) would be considered physiologic, and you wouldn’t worry too much about it (other than you’d want to know why the patient was so anemic). In other settings, however, an LEBR is more ominous. Unless there’s a damn good reason for the bone marrow to be pouring immature hematopoietic cells into the blood (like, as we just discussed, a very low hemoglobin), then what you’d worry about is that there is something filling up the marrow – like carcinoma, or leukemia, or fibrosis – that is pushing those precursors out into the blood prematurely. Fully two-thirds of LEBRs occur in this second, scary scenario. Use the hemoglobin to help differentiate between scary and non-scary causes. If the hemoglobin is below 6, an LEBR is probably physiologic; if it’s over 6, make sure there isn’t something else going on.

LEBR + Hgb6: could be bad

Things to make you look smart Q. Why is a bunch of immature neutrophils in the blood called a left shift? A. It’s called a left shift because in the olden days, when people counted cells in the blood, they didn’t have any electronic gizmos; they just tallied them by hand in columns. By convention, the earliest precursors (myeloblasts) were listed on the left, and then the rest of the precursors in order after that, with segmented neutrophils farthest on the right. So in a normal blood smear, all the marks would be in the right-most column, but if there were immature neutrophils present, the marks would get “shifted” to the left.

www.pathologystudent.com

The Complete (but not obsessive) Hematopathology Guide page 41

Benign Lymphocytosis Where do lymphocytes live? Most lymphoid stem cells in adults are in the bone marrow. Growth factors like IL-2 induce differentiation into T, B, and NK cell precursors, which then travel to different organs (e.g., lymph nodes, spleen, thymus, mucosa-associated lymphoid tissue) for further processing and storage.

Normal lymphocyte count Unlike the normal neutrophil count, which is pretty much the same no matter how old you are, the normal lymphocyte count varies with age. It’s highest right after birth (at 2 weeks: 2.0 - 17.0 x 109/L), and declines with age (at age 4: 2.0 - 8.0 x 109/L; by age 18: 1.0 - 4.0 x 109/L). Another important parameter is the immunophenotype of the lymphocytes. Normally, we have way more T cells than anything else (about 60-80% of our lymphocytes are T cells; 10-20% are B cells, and 510% are NK cells). That can be useful in telling a benign lymphocytosis from a malignant one, because cells in lymphoid malignancies are all the same type (all B cells or all T cells).

Mature lymphocytosis in Bordetella pertussis infection

Proliferation of mature lymphocytes Benign lymphocytosis has two flavors: mature and reactive. If all you see are a bunch of matureappearing lymphocytes, then the top three benign choices are infectious lymphocytosis, whooping cough, and transient stress lymphocytosis. Infectious lymphocytosis is a viral disorder of childhood in which the lymphocyte count can climb as high as 100 x 109/L. Whooping cough (caused by Bordetella pertussis infection) usually shows a moderate lymphocytosis (up to 55 x 109/L). Transient stress lymphocytosis is a temporary phenomenon seen in adults with emergency medical conditions, such as cardiac arrest. The count generally does not exceed 10 x 109/L, and it generally falls back to normal within a day or two.

www.pathologystudent.com

Mature lymphocytosis: infectious lymphocytosis, Bordetella pertussis, or stress lymphocytosis

The Complete (but not obsessive) Hematopathology Guide page 42

Proliferation of reactive lymphocytes If, on the other hand, the lymphocyte count is elevated and you see lots of funny-looking lymphocytes, you may be dealing with a reactive lymphocytosis. There are several kinds of “reactive” lymphocytes, which are simply benign lymphocytes that have a different appearance than the normal, mature lymphocytes you typically see in the blood. One group of reactive lymphocytes is termed “Downey cells,” after the hematologist Hal Downey who described them in 1923. Downey described three types of cells commonly found in infectious mononucleosis: a small lymphocyte with lobed nucleus and scant, compact cytoplasm (later termed the Downey I cell); a large, “fried-egg” lymphocyte with copious cytoplasm containing radial striations (later termed the Downey II cell), and a very large lymphocyte with a large nucleus showing a fine chromatin pattern (later termed the Downey III cell). Downey cells are seen in infectious mononucleosis but also in pediatric viral infections such as measles, mumps, rubella and chickenpox. Other reactive lymphocytes include plasma cells and their precursors (like proplasmacytes, cells that are almost to the plasma cell stage, but don’t yet have nice clock-face chromatin or a welldeveloped hof), and plasmacytoid lymphocytes (cells that have an eccentric nucleus and a hof, like plasma cells, but with clumpy-smudgy lymphocyte chromatin rather than the blocky clock-face chromatin of a plasma cell). These reactive lymphocytes are seen in pediatric childhood infections, viral hepatitis, and various immune disorders such as autoimmune diseases and drug reactions.

Proplasmacyte (L) and Downey III cell (R)

Things to make you look smart Q. What’s the significance of the different types of Downey lymphocytes? A. Of the three types of Downey cells, the one most commonly seen in infectious mononucleosis is the type II cell (the one that looks like a fried egg). However, you can see this cell in other benign viral disorders too. The type of Downey cell that is the most specific for infectious mononucleosis is the type III cell (the huge one with fine chromatin). If you see this one, you can be pretty darn sure that the patient has mono.

www.pathologystudent.com

The Complete (but not obsessive) Hematopathology Guide page 43

Differentiating Benign from Malignant Leukocytoses In real life, a patient does not come to you and say, “I have a benign lymphocytosis. Please tell me what the cause is.” What happens is that you get a blood smear, and you have to make sure it’s not malignant (then you can go about your business of figuring out which benign disorder you’re dealing with). So let’s talk about how you would differentiate between these benign things and a couple malignant things that could look very similar.

Left shift vs. CML The left shift that we talked about can look very similar to a malignant disorder known as chronic myeloid leukemia (or CML), discussed on page 65. Patients with CML have a very high white count in the blood, and virtually all the cells are neutrophils in varying stages of development. So how would you tell the difference between the two?

CML vs. left shift:

Philadelphia chromosome

Well, patients with CML generally have more immature cells (like myelocytes and promyelocytes) than patients with a benign left shift (in which you see a lot of bands and metamyelocytes, but fewer of the earlier precursors like promyelocytes). In other words, the shift to the left is usually more extreme in CML than it is in a benign left shift. Also, the white count itself is usually higher in CML (on the order of 50,000 – 100,000) than it is in a benign left shift (where the count doesn’t usually go above 25,000 or so). Those are pretty soft criteria, though. Something more concrete is basophilia, a finding that is virtually always present in CML, for some weird reason. Finally, if you really want to be definitive, all cases of CML have a chromosomal translocation between chromosomes 9 and 22, producing what’s known as the Philadelphia chromosome. Benign left shifts do not have this.

Mature lymphocytosis vs. CLL In the lymphoid category, the benign mature lymphocytosis we talked about earlier can look identical to a malignant lymphoid disorder called chronic lymphocytic leukemia (or CLL), discussed on page 72. Patients with CLL have an elevated lymphocyte count, and all the lymphocytes look mature. In fact, if all you had was a blood smear of each disorder, you’d be very hard pressed to tell them apart. What’s a person to do?

CLL vs. mature lymphocytosis:

flow

One thing to do would be to look at the age of the patient, if you have that information. Most cases of mature lymphocytosis occur in kids, and all (yes, you read that right, all) cases of CLL occur in adults. Another thing to do would be flow cytometry. In a benign mature lymphocytosis, there would be a mixture of B and T cells, but in CLL, all the cells mark as B cells (and, strangely, they also express CD5, a marker traditionally thought of as a T cell marker).

www.pathologystudent.com

The Complete (but not obsessive) Hematopathology Guide page 44

Other Leukocytoses Monocytosis The normal monocyte count is somewhere around 0.3 - 0.5 x 109/L. An elevated monocyte count may be associated with solid malignancies, autoimmune disease, and infection.

Basophilia The normal basophil count is between 0.01 - 0.1 x 109/L. There is only one real cause of basophilia, and that’s chronic myeloid leukemia. You may see iron-deficiency anemia as a cause in board review books, but they’re full of crap (at least as far as basophilia goes). Any time you see a basophilia, you must rule out CML, even if the patient has no other symptoms or signs, because basophilia is one of the earliest signs of CML!

Eosinophilia The normal eosinophil count is 0.05 - 0.3 x 109/L. Some fairly frequent causes of eosinophilia include drug allergies, asthma, and skin diseases. Rarely, a patient with eosinophilia will be found to have intestinal parasitism (but for the frequency of test questions on this particular fact, you’d think it happened all the time!). Other rare causes include chronic ulcerative colitis, hepatitis and sarcoidosis .

www.pathologystudent.com

Monocytosis

The Complete (but not obsessive) Hematopathology Guide page 45

Four. Leukemia Introduction to Hematologic Malignancies Before we get into our discussion of leukemia, let’s take a moment to review how hematopoietic malignancies are organized.

Leukemia vs. lymphoma There are two main types of hematopoietic malignancies: leukemias and lymphomas. Leukemias are hematopoietic malignancies that begin in the bone marrow. They almost always spill over into the blood, and sometimes they involve other tissues too, like lymph nodes, spleen or CNS. Lymphomas are hematopoietic malignancies that start in lymph nodes. They may spill over into the blood, and sometimes they involve other tissues, like bone marrow, spleen or CNS. Leukemias and lymphomas have their own classification schemes, and they are discussed separately. However, there are many leukemia/lymphoma pairs that are now known to be the same diseases. For example, chronic lymphocytic leukemia is now known to be the same disease as small lymphocytic lymphoma (both have the same morphology, immunophenotype and prognosis), and they are now lumped together as “CLL/SLL.” We’ll point out these pairings as we go along.

Leukemia:

bone marrow

Lymphoma:

lymph nodes

Myeloid vs. lymphoid Besides the leukemia/lymphoma division, there is another big division in hematopoietic malignancies, namely, that between lymphoid and myeloid malignancies. Lymphoid cells include lymphocytes and their precursors. Myeloid cells include, well, everything else (neutrophils, eosinophils, basophils, monocytes, red cells, and platelets). By the way, the word myeloid means “of the marrow” – so basically everything that grows and lives in the marrow (that is, every hematopoietic cell except lymphocytes) is considered myeloid. Just to make things more interesting, sometimes people use the term myeloid to refer to cells of the neutrophil lineage. Confusing, isn’t it? So if someone says “myeloid,” you need to figure out whether they’re talking about all non-lymphoid cells or just neutrophils.

www.pathologystudent.com

The Complete (but not obsessive) Hematopathology Guide page 46

Summarizing the diseases in your head Here’s a couple of ways you may want to group these diseases together:

The Leukemia-vs.-Lymphoma Grouping

The Myeloid-vs.-Lymphoid Grouping

Leukem ias

Myeloid malignancies • Acute myeloid leukemia • Chronic myeloproliferative disorders • Myelodysplastic syndromes

Acute leukem ias • Acute myeloid leukemia • Acute lymphoblastic leukemia Chronic leukemias • Chronic myeloproliferative disorders • Chronic lymphoproliferative disorders M yelodysplastic syndrom es Lym phom as • Non-Hodgkin lymphoma • Hodgkin disease Plasm a cell disorders •

Lym phoid m alignancies B-cell m alignancies • Acute lymphoblastic leukemia, B-cell type • Non-Hodgkin lymphoma, B-cell types • Myeloma T-cell m alignancies • Acute lymphoblastic leukemia, T-cell type • Non-Hodgkin lymphoma, T-cell types Hodgkin disease

Myeloma

www.pathologystudent.com

The Complete (but not obsessive) Hematopathology Guide page 47

Acute Leukemias As you’ll see in the next two sections, there is a big conceptual difference between the acute and chronic leukemias. They are very different clinically (acute leukemias have a sudden, tumultuous onset; chronic leukemias present more insidiously) and morphologically (acute leukemias have tons of blasts; chronic leukemias have more mature cells). It will be helpful if you keep these generalizations in mind as we talk about the different types of leukemia. Otherwise it can be easy to get lost in the trees and miss the forest.

Pathophysiology The acute leukemias are malignant, monoclonal proliferations of immature myeloid or lymphoid cells in the bone marrow. That’s the official definition. If you want a quick and dirty definition, it’s when the marrow (and usually the blood) fills up with malignant blasts. The process is caused by two things: clonal expansion (one crappy little cell divides into two, and those divide into four, and then 16, and then 32, etc. etc. until you have a huge population of cells all descended from one bad little cells) and maturation failure (where instead of growing and maturing, cells stay stuck at one stage of development). It’s not a great idea to have these malignant cells around for many different reasons. One big problem is something called “bone marrow failure.” If you have a tone of cells of any type filling up the marrow space, then there’s not going to be much room for normal red cells, white cells and platelets to grow – so their counts will go down in the blood, and there won’t be enough of them to do their respective jobs. Sometimes, the malignant cells themselves can release factors that inhibit growth and function of normal hematopoietic cells. Finally, if the malignant cells leave the bone marrow space, they can infiltrate and damage other organs (liver, spleen, brain, others).

Bone marrow failure:

Clinical Features Acute leukemias come on suddenly, over a period of days or at most weeks. Usually, the presenting symptoms are related to bone marrow failure. Patients get fatigued and tachycardic (from the anemia); they may have recurrent infections (from the decreased numbers of normal white cells); and they may have excessive bleeding or bruising (from the thrombocytopenia). Some patients also get bone pain (from the rapidly-expanding mass of malignant cells in their bones), or symptoms of extramedullary involvement (like lymphadenopathy, hepatosplenomegaly, or CNS infiltration).

www.pathologystudent.com

Blood full of blasts in acute leukemia

few or no normal hematopoietic cells in bone marrow

The Complete (but not obsessive) Hematopathology Guide page 48

Acute Myeloid Leukemia Acute myeloid leukemia (AML) is a malignant proliferation of myeloid blasts (or very early myeloid precursors) in the bone marrow and blood. By the way, “myeloid” here means anything but lymphoid. So the myeloid blasts might be myeloblasts (that would turn into neutrophils), or monoblasts, or erythroblasts, or megakaryoblasts.

To make a diagnosis of AML, need at least 20% blasts

The blasts (or very early myeloid precursors) must comprise at least 20% of all the nucleated cells in the blood and/or bone marrow to make the diagnosis of AML (if only 10% of all the nucleated cells are blasts, for example, then you can’t call it AML).

C lassification There are lots of different kinds of AML. Some have unique genetic abnormalities. Some have monoblasts, or erythroblasts, or something else instead of myeloblasts. Some occur as a result of previous chemotherapy. All of these different types have to be classified in some way that makes sense for therapy and prognosis. Right now, we’re transitioning between two different classifications. The old classification (called the FAB, or French-American-British classification) was based mostly on morphology (how the cells look). It labels the different types of AML with numbers (AML-M0, AML-M1, AML-M2, etc.). The new classification (created by the World Health Organization), takes into account a lot of different things, such as morphology, genetic changes, and whether the leukemia is related to previous chemotherapy. The new WHO classification is more helpful than the old FAB one for determining treatment and prognosis. However, you’ll definitely see references to the old FAB classification from time to time.

WHO Classification of Acute Myeloid Leukemia 1. AML with genetic abnormalities • t(8;21) • inv(16) • t(15;17) • 11q23 2. AML with an FLT3 mutation 3. AML with multilineage dysplasia 4. AML and myelodysplasia, therapy-related 5. AML not otherwise characterized

www.pathologystudent.com

The Complete (but not obsessive) Hematopathology Guide page 49

How to Make a Diagnosis Morphology In AML, you’ll see a lot of blasts (or very early myeloid precursor cells) in the blood and bone marrow. You have to count them, though, because there’s a cutoff you need to make in order to call it AML: at least 20% of all nucleated cells (that means everything except mature red cells) in the blood or bone marrow must be myeloblasts. Some AMLs, as we’ll see in a minute, don’t have a ton of myeloblasts, but instead have a ton of some other really early myeloid precursor. In acute promyelocytic leukemia, for example, the proliferating malignant cell is the promyelocyte (there aren’t that many myeloblasts around). In those types of AML, that particular early precursor cell is substituted for the myeloblast when determining whether the case makes the 20% cutoff. So for acute promyelocytic leukemia, if more than 20% of the nucleated cells in the blood or bone marrow are promyelocytes, that’s enough to call it acute promyelocytic leukemia (you don’t need to worry about myeloblasts). In case you need a little refresher as to what different cells look like, here’s a list of the cells that can be more difficult to distinguish: Myeloblast Large nucleus with fine chromatin, nucleoli. Thin rim of basophilic cytoplasm. May have a few fine, azurophilic, cytoplasmic granules. Prom yelocyte Biggest cell in myeloid line. Lots of primary (coarse, azurophilic) granules. Monoblast Typical blast cell but larger, with more oval-shaped nucleus. Prom onocyte Delicate "tissue-paper" folds in nucleus. Monocyte Large cell with grayish ("dishwater") cytoplasm, "raked" chromatin. In some cases of AML, there is pretty much nothing but blasts. Everywhere you look, just blasts; no differentiating cells at all. Those can be difficult to diagnose. How do you even know if it’s a myeloid leukemia? There are a couple of morphologic clues that will help you place the leukemia in the myeloid category: dysgranulopoiesis and Auer rods Dysgranulopoiesis means “disordered neutrophil production” (translation: funny-looking neutrophils). Usually, even if you have a sea of blasts, if you look long enough you can find a mature neutrophil here and there. If those neutrophils are strange-looking, that’s a clue that the blasts are myeloid. Sometimes the neutrophils will not have enough granules in the cytoplasm (“hypogranularity”); sometimes instead of multiple nuclear lobes, you’ll just see two, or even one (“hyposegmentation”).

www.pathologystudent.com

The Complete (but not obsessive) Hematopathology Guide page 50

A more definitive clue is the presence of Auer rods. Auer rods are needle-like, eosinophilic inclusions in the cytoplasm of malignant myeloblasts. They are composed of secondary granules lined up all in a row – but they look more like toothpicks than collections of granules. Auer rods are only seen in malignant cells of the neutrophil series (and virtually always, they occur at the myeloblast stage of development). So – if you find a blast in that sea of blasts that has an Auer rod in it, you know for sure that a) it is a myeloblast, and b) it is malignant. And by extension, the rest of the blast population is myeloid as well. A note of caution: just because you don’t see Auer rods, that doesn’t mean you can rule out AML! There are lots of cases of real AML in which you don’t see Auer rods. So: if you see them, it’s AML. If you don’t see them, it could be either AML or ALL. Cytochemical stains Sometimes, you have a ton of blasts, and no dysgranulopoiesis and no Auer rods. Then what? Well, you’ll probably be sending the specimen off for other tests (like Immunophenotyping and cytogenetics). But those tests take a while to perform. In the meantime, there are a few stains you can do in the lab, right on an unstained smear, that can be very helpful.

Blast with two Auer rods

Myeloperoxidase (MPO) is a stain that highlights neutrophils and their precursors (it also highlights eosinophils, but a leukemia of eosinophilic cells is exceedingly rare, so this isn’t usually an issue). So, if you have a ton of blasts, and they light up with myeloperoxidase staining, you know they are myeloblasts. Sudan black B (SBB) does the same thing; it’s just not used as frequently as MPO. Another commonly-used cytochemical stain is non-specific esterase (NSE). This stain highlights monocytes and their precursors. So the same reasoning applies: if you have a ton of blasts, or blast-like cells, and they light up with NSE, you know they are of the monocytic lineage (probably either monoblasts or promonocytes). Immunophenotyping As mentioned on page 11, immunophenotyping takes a look at the markers on the surface of the cells. This is of great use when you don’t know what a cell is! Common myeloid markers include “pan-myeloid” markers such as CD33 (that stain virtually all myeloid cells) and more specific myeloid markers, such as CD61 (which stains only megakaryoblasts). One thing to be aware of, though, is that acute myeloid leukemias can sometimes aberrantly express lymphoid markers. Just to make things more difficult.

www.pathologystudent.com

Monoblasts and a monocyte (L) positive for NSE

The Complete (but not obsessive) Hematopathology Guide page 51

Cytogenetics/Molecular Studies Several subtypes of AML have specific chromosomal abnormalities. Compared to other types of AML, these subtypes are treated differently and have different prognoses. The most important abnormalities are: t(8;21) (better prognosis) t(15;17) (better prognosis) • inv(16) (better prognosis) • 11q23 (worse prognosis) • AML with FLT3 mutation (worse prognosis) The only way to detect these, of course, is to do cytogenetic or molecular assays. • •

AML with genetic abnormalities Now let’s take a look at the different types of AML. We’ll use the WHO classification as our overall guide, but we’ll also throw in the FAB subtypes when appropriate. That way you’ll have heard of everything, and you’ll be super smart no matter what classification people throw at you. The first subtype is AML with genetic abnormalities. Cases of AML in this category have – you guessed it – specific genetic abnormalities. These abnormalities confer either a better or worse prognosis, and the leukemias are treated with different drugs. There are only four abnormalities in this category: t(8;21), t(15;17), inv(16), and 11q23. Mutation of the FLT3 gene is also very significant – but that genetic abnormality gets its own category.

good = t(8;21), t(15;17), inv(16) bad =11q23, FLT3

AM L with t(8;21) Cases of AML with a t(8;21) usually have the morphology of an AML-M2 in the old FAB classification (a lot of myeloblasts, with some maturing neutrophilic cells too). Prognosis is relatively favorable. AM L with inv(16) Cases of AML with an inv(16) usually have the morphology of an AML-M4 in the old FAB classification (lots of myeloblasts and neutrophil precursors, but also lots of monocytic cells). In addition, there are some unusual morphologic findings in these cases – most notably, the presence of huge eosinophils with deep purple granules (weird). These cases are sometimes called “AML-M4 Eo” because of the abnormal eosinophils. Prognosis is relatively favorable.

www.pathologystudent.com

The Complete (but not obsessive) Hematopathology Guide page 52

AM L with t(15;17) All cases of AML with a t(15;17) have the same morphology: they fall into the category of acute promyelocytic leukemia (or AML-M3 in the FAB classification). The marrow and blood are full of promyelocytes (at least 20%, but usually more). These (malignant) promyelocytes look kind of like benign promyelocytes, but they tend to have way more granules. One pathognomonic finding in this type of AML is the presence of promyelocytes with TONS of Auer rods in them (like 40 or 50). These cells are called “faggot cells” (faggot meaning “bundle of sticks”). Patients with acute promyelocytic leukemia are at great risk for DIC (disseminated intravascular coagulation) if given conventional treatment. The granules inside the promyelocytes have procoagulant substances in them – so if you bust open the cells (as normal chemotherapeutic agents do), the patient is at great risk for clotting. This made treatment difficult before we understood what was going on at the genetic level in these cells. It turns out that the t(15;17) encodes for an abnormal retinoic acid receptor which blocks cell differentiation. A drug called ATRA (all-trans retinoic acid) is now used for therapy in these patients. It acts as a differentiating agent, allowing the screwed-up promyelocytes (with the screwed-up retinoic acid receptor) to mature into neutrophils (and then die normally). Then, with most of the dangerous granule-containing promyelocytes gone, you can treat the patient with cell-busting chemotherapy. Prognosis is now favorable in these cases.

Faggot cell in APL

AM L with 11q23 abnorm alities This type of AML usually has a monocytic component (most cases resemble either AML-M4 or AML-M5 in the FAB classification). Two groups of patients are more likely to get this type of AML: infants and patients who have been treated in the past with chemotherapy (for another tumor). Unfortunately, in this type of AML, prognosis is poor (even more poor than the regularly poor prognosis of AML).

AML with FLT3 mutation This type of AML has a mutation of FLT3 (duh), which is short for FMS-like tyrosine kinase-3. This mutation is present in almost a third of cases of AML (wow!), making it the most common genetic abnormality in AML. Most of these cases have a monocytic component (meaning they would have been called AML-M4 or M5 using the FAB classification). Patients with AMLs with this mutation have an increased relapse rate and shortened survival; their overall prognosis is poor.

www.pathologystudent.com

The Complete (but not obsessive) Hematopathology Guide page 53

AML with multilineage dysplasia AMLs that fall into this category have two things going on: 1. At least 20% blasts in blood or bone marrow 2. Dysplasia in at least two myeloid cell lines, including some of the following: • Dysgranulopoiesis (neutrophils with hypogranular cytoplasm, hyposegmented nuclei, or bizarrely segmented nuclei) • Dyserythropoiesis (megaloblastic nuclei, karyorrhexis, or multinucleation of erythroid precursors; ringed sideroblasts) • Dysmegakaryopoiesis (micromegakaryocytes, and normal-to-large sized megakaryocytes with monolobed nuclei or multiple separated nuclei) This type of AML often occurs in elderly patients; a frequent presentation is severe pancytopenia. It may occur de novo or may evolve from a myelodysplastic syndrome (see page 59) or a myeloproliferative disorder (see page 65). Patients often have chromosome abnormalities (especially of chromosomes 5 and/or 7). It is hard to get patients with this type of AML into remission, and the overall prognosis is poor.

AML, therapy-related This category includes both AML and myelodysplastic syndrome (MDS) that occurs as a result of previous chemotherapy and/or radiation therapy. Most cases of therapy-related AML or MDS are caused by one of two classes of chemotherapy drugs: alkylating agents and topoisomerase II inhibitors. AM L/M DS caused by alkylating agents This type of AML usually occurs about 5-6 years after exposure to an alkylating agent, such as busulfan, cyclophosphamide and ifosfamide. It usually starts out as a myelodysplastic syndrome, and is often accompanied by bone marrow failure. Many patients die in the MDS phase, but in some patients, the disease evolves into AML. Cytogenetic abnormalities are almost always present (usually involving chromosomes 5 and 7). AM L/M DS caused by topoisom erase II inhibitors This type of AML usually occurs about 2 years after exposure to topoisomerase II inhibitors, such as epipodophyllotoxins (e.g., etoposide) or anthracyclines (e.g., doxorubicin). There is usually a prominent monocytic component (most cases would be called M4 or M5A in the FAB classification). Cytogenetic abnormalities are frequently present, and usually involve chromosome 11q23 and the MLL gene.

www.pathologystudent.com

The Complete (but not obsessive) Hematopathology Guide page 54

AML, not otherwise classified Cases that don’t fit into one of the other four WHO categories above are placed into this category. This is a good place to discuss the FAB classification subtypes, many of which did not come into our discussion above. You’re going to run into these names, so might as well learn a bit about them.

FAB Classification of Acute Myeloid Leukemia AML-M0 – Acute myeloblastic leukemia, minimally differentiated AML-M1 – Acute myeloblastic leukemia without maturation AML-M2 – Acute myeloblastic leukemia with maturation AML-M3 – Acute promyelocytic leukemia AML-M4 – Acute myelomonocytic leukemia AML-M5 – Acute monocytic leukemia AML-M6 – Acute erythroid leukemia AML-M7 – Acute megakaryoblastic leukemia

AM L-M 0 (AML, m inim ally differentiated) This type of AML has a ton of myeloblasts in the blood and bone marrow (usually myeloblasts comprise over 90% of the nucleated cells). However, as the name says, the myeloblasts show minimal differentiation. This means that the malignant myeloblasts are stuck in such an early stage of development that you almost can’t even tell whether they are myeloid or lymphoid! They don’t have Auer rods, and they don’t stain for MPO. So you need to use immunophenotyping to figure out that the cells are myeloblasts. AM L-M 1 (AML without m aturation) This type of AML also has a ton of myeloblasts in the blood and bone marrow (usually myeloblasts comprise over 90% of the nucleated cells). However, in contrast to the myeloblasts in AML-M0, these myeloblasts do show some differentiation. You might see a few Auer rods, and some of the blasts will stain positively for MPO. As the name says, there is no maturation. Which means you only see myeloblasts – nothing more mature (like promyelocytes or myelocytes).

www.pathologystudent.com

M0 = # myeloblasts

M1 = # myeloblasts

(How’s it different from M0?)

The Complete (but not obsessive) Hematopathology Guide page 55

AM L-M 2 (AML with m aturation) In this type of AML, there are enough myeloblasts to make the 20% cutoff, and they’re differentiated (meaning you see signs of myeloid-ness, like Auer rods and MPO staining). However, in contrast to AML-M1, there is maturation. In addition to the myeloblasts, you’ll see all the other stages of neutrophil development (promyelocytes, myelocytes, metamyelocytes, and segmented neutrophils). All of these cells are part of the same malignant clone – it’s just that the malignant cells are not stuck in the myeloblast stage; they are able to mature into segmented neutrophils.

M2 = # myeloblasts + maturing neutrophils

As mentioned above (page 52), some cases of AML-M2 show a t(8;21). Patients with this type of AML receive different chemotherapeutic agents, and have a better prognosis. You can often tell when a case of AML-M2 has a t(8;21), even before you get the cytogenetic report back, because of the peculiar way these cases look. The myelocytes often have a coalescence of specific granules in the cytoplasm (which looks like a big pink cytoplasmic blob), as well as extra-long, needle-like Auer rods. Weird. AM L-M 3 (Acute promyelocytic leukemia) This type of AML is characterized by a proliferation of malignant promyelocytes. So to make the diagnosis, you don’t need to worry about myeloblasts; you just need to make sure that the promyelocyte population comprises >20% of the nucleated cells. All (or virtually all) of these cases of AML have a t(15;17) which, as discussed on page 53, results in the formation of an abnormal retinoic acid receptor that interferes with the maturation of the cells (they get stuck in the promyelocyte stage, and can’t mature any further). It’s cool that the initial treatment (all-trans retinoic acid, or ATRA) targets this messed-up receptor, allowing the cells to differentiate. These patients have a relatively good prognosis. M4 (Acute myelom onocytic leukem ia) AML-M4 is kind of a dual-lineage leukemia: you see lots of myeloblasts, but also a lot of monocytic cells (monoblasts and promonocytes). Combined together, all these must equal >20% of the nucleated cells in the blood or marrow.

M3 = # promyelocytes

M4 = # promyelocytes

As mentioned above (page 52), some cases of AML-M4 have an inv(16). These cases usually have an unusual morphology, in which there are lots of big eosinophils with dark granules. This variant is called AML-M4 Eo, and it has a better prognosis than AML in general.

www.pathologystudent.com

The Complete (but not obsessive) Hematopathology Guide page 56

M5 (Acute monocytic leukem ia) AML-M5 comes in two flavors: AML-M5A and AML-M5B. AML-M5A has lots of monoblasts, and AML-M5B has lots of promonocytes. There may be a mixture of blasts (with some myeloblasts thrown in) – so the way it’s worded is: at least 20% of the cells must be blasts. If 80% or more of those are monoblasts, it’s an M5A; if 80% or more are promonocytes, it’s an M5B. AMLs with a monocytic component (that is, M4 or M5) have an increased incidence of extramedullary involvement. Common sites are skin, central nervous system, gums, and testes. Patients with these types of AML are often given different treatment (for example, chemotherapeutic agents are injected into the CSF) to help prevent this type of spread.

AM L-M 6 (acute erythroblastic leukem ia) This type of AML involves both the myeloid and erythroid lineages. There are lots of myeloblasts (at least 20%) and also lots of erythroid precursors (usually these are greater than 50% of all nucleated cells). The erythroid precursors may be giant and bizarre, with bilobed nuclei and multinucleation.

AML-M5B

M5A = # monoblasts M5B = # promonocytes

AM L-M 7 (acute m egakaryoblastic leukem ia) This one is just what the name says: an AML that involves the megakaryocytic lineage. The blood and bone marrow show at least 20% megakaryoblasts. You’d think these would look weird (because megakaryocytes look so weird) – but they often just look like bland, run-of-the mill blasts. There aren’t any cytochemical stains that are specific for megakaryocytes or megakaryoblasts – so usually you need to resort to flow cytometry. Cells of the megakaryocytic lineage usually express CD41 and/or CD61.

www.pathologystudent.com

M6 = # erythroblasts + myeloblasts M7 = # megakaryoblasts

The Complete (but not obsessive) Hematopathology Guide page 57

Treatment and Prognosis Patients with AML generally get a big dose of chemotherapy to start with. It differs depending on the type of AML (patients with acute promyelocytic leukemia, for example, receive ATRA, as mentioned above). About two-thirds of patients enter a complete remission (meaning there’s no evidence of disease) after chemotherapy. Unfortunately, this remission is often short-lived. Patients who have a bone marrow match, and who are able to tolerate transplant (meaning they are relatively young and relatively healthy) are usually taken to bone marrow transplantation. This is really the only chance for a cure for AML at this point – but the procedure is not without risk (particularly infection). Even with bone marrow transplantation, prognosis is dismal. Overall, about 80% of patients die within 3 years. The prognosis varies inversely with age (older patients do worse). Cytogenetics are important in determining prognosis. Patients with t(8.21), inv(16) and t(15;17) survive longer; patients with FLT-3 mutations or 11q23 abnormalities have a worse prognosis.

www.pathologystudent.com

The Complete (but not obsessive) Hematopathology Guide page 58

Myelodysplastic Syndromes (MDS) The myelodysplastic syndromes are a group of disorders characterized by dysplastic changes in myeloid cells, with or without an increase in blasts. MDS occurs more frequently in older patients (like around 60-70). It may be asymptomatic, or it may present with symptoms of bone marrow failure (weakness, infections, bleeding). Many – but not all – cases evolve into acute leukemia.

MDS = dysplasia ± # blasts

Classification There are currently 6 types of MDS, and though we won’t get into the particular requirements for diagnosis (that’s more for hematopathology boards), you can tell from the name what each one involves: refractory anemia (RA), refractory anemia with ringed sideroblasts (RARS), refractory anemia with excess of blasts (RAEB), chronic myelomonocytic leukemia (CMML), chronic myelomonocytic leukemia in transformation (CMML-T), myelodysplastic syndrome, unclassified (MDS-U).

Morphology As mentioned above, myelodysplastic syndromes are characterized by dysplastic changes in myeloid cells. These changes look different in each cell line: • Red cell dysplasia - nuclear lobulation and fragmentation. • Neutrophil dysplasia - hypogranulation, hyposegmentation • Megakaryocytic dysplasia - small megakaryocytes with non-lobulated nuclei In addition to the dysplastic changes, you may see increased numbers of blasts. However, remember that if you see > 20% blasts, you must classify the case as acute leukemia!). One other morphologic finding of note: most patients with MDS present with a macrocytic anemia. Remember the other macrocytic anemias we talked about? If not, see pages 17 and 36. If you have a patient with a macrocytosis (MCV >100), particularly if your patient is an older patient, MDS is something you need to consider.

Treatment and Prognosis Some types of MDS (like refractory anemia) tend to be lower-grade, meaning that they rarely evolve into acute leukemia. These are treated supportively (with transfusions if necessary – but no big time chemotherapy). The course in these cases is long, and the prognosis is relatively good. Other types of MDS (like refractory anemia with excess of blasts) tend to be more aggressive, with rapid evolution to AML in about one third of the patients. These cases are treated with aggressive anti-leukemic therapy.

www.pathologystudent.com

The Complete (but not obsessive) Hematopathology Guide page 59

Acute Lymphoblastic Leukemia Acute lymphoblastic leukemia (ALL) is a malignant proliferation of lymphoblasts in the bone marrow and blood. In ALL, there’s no 20% blast cutoff for diagnosis like there is in AML. Usually, there’s way more than that – often most of the nucleated cells in the bone marrow and blood are blasts. ALL is the most common type of leukemia in children (about 80% of all childhood leukemias are ALL). Most cases occur in patients under 15 years of age, but a small number of cases occur in adults. The prognosis is relatively good for children, but poor for adults.

ALL is the most common

leukemia in children.

C lassification ALL is classified according to immunophenotype. It’s pretty straightforward: TdT

T-cell Ag

B-cell Ag

T-cell ALL

+

+

-

Precursor-B ALL

+

-

+

Burkitt leukemia

-

-

+

The two big divisions are between T-lineage and B-lineage leukemias. The T-lineage leukemias are positive for TdT (terminal deoxynucleotidal transferase, an enzyme present in lymphoblasts) and for T-cell antigens (like CD3). The B-lineage leukemias are divided into precursor-B ALL (which is positive for TdT and B-cell antigens (like CD21), and Burkitt leukemia (or B-cell ALL) (which is positive for B-cell antigens, but negative for TdT). Let’s look at the different types.

T-cell ALL T-cell ALL has markers that are really easy to remember (TdT positive, T-cell-antigen positive). It accounts for about 15% of all cases of ALL, and it’s more common in the teenage years (perhaps because that’s the age at which the thymus reaches its maximal size). It often presents with a mediastinal mass and a very high white blood count. Prognosis has traditionally been poor, but with intensive chemotherapy, prognosis now approaches that of precursor-B ALL. It is the same disease as T-lymphoblastic lymphoma.

www.pathologystudent.com

T-cell ALL = T-lymphoblastic lymphoma

The Complete (but not obsessive) Hematopathology Guide page 60

Precursor-B ALL Precursor-B ALL also has markers that are easy to remember (TdT positive, B-cell-antigen positive). It’s by far the most common type of ALL, accounting for about 80% of all cases. Here’s another leukemia/lymphoma pairing: B-cell precursor ALL is the same thing as B-lymphoblastic lymphoma, which we’ll discuss later. The prognosis is pretty good, especially in children.

Precursor-B ALL =

B-lymphoblastic lymphoma

Burkitt leukemia Burkitt leukemia (or, as it was known before the latest classification change, B-cell ALL) has a weird immunophenotype: it’s TdT negative! Weird, considering it’s a leukemia composed of lymphoblasts, and TdT is a lymphoblast marker. Whatever. The morphology is unusual too: the lymphoma cells are big cells with deep blue cytoplasm and sharply punched-out vacuoles. Most cases have a translocation between chromosomes 8 and 14, involving the c-myc gene on chromosome 8 and the heavy chain gene on chromosome 14. Sometimes, instead of chromosome 14, chromosomes 2 or 22 are involved. The kappa light chain gene is on chromosome 2 and the lambda light chain gene is on chromosome 22 (it’s easy to remember: the earlier letter goes with the smaller number).

Burkitt leukemia = Burkitt lymphoma

Burkitt leukemia accounts for about 1% of all cases of ALL; the prototypic patient is an older child with a fast-growing abdominal mass. Burkitt leukemia is the same disease as Burkitt lymphoma (the official name is: “Burkitt leukemia/lymphoma”). Whatever you want to call it, it’s an extremely fast-growing tumor. Which means it has this paradox of being aggressive but also relatively sensitive to chemotherapy (traditional chemotherapy hits dividing cells – so if a tumor has a lot of dividing cells, it’s more sensitive to chemotherapy).

How to make a diagnosis Morphology In ALL, there are lots of blasts in the blood and bone marrow. Remember in AML how you had to count the blasts and make sure that they numbered at least 20% of the nucleated cells in the blood or bone marrow? Well, as mentioned above, that doesn’t hold for ALL. There’s no cutoff. You do count the cells, because it’s important to know how heavy the tumor burden is – but the presence of even a small number of lymphoblasts is enough to make the diagnosis. In many cases, the marrow and blood are totally stuffed full of lymphoblasts, and counting is kind of a moot point.

www.pathologystudent.com

Burkitt leukemia

The Complete (but not obsessive) Hematopathology Guide page 61

Lymphoblast morphology is not very exciting. Some lymphoblasts are large, with big nuclei and fine chromatin; others are smaller with more condensed chromatin. In ALL, it is often hard to tell whether the blasts are lymphoid or myeloid just by looking at them under the microscope, because usually lymphoblasts are pretty non-descript. Unlike the myeloblasts in AML, which may have Auer rods in them, the lymphoblasts in ALL do not have any definitive distinguishing characteristics. One soft clue that you’re dealing with a lymphoid (as opposed to myeloid) process is the presence of “ghost” or “basket” cells. These are remnants of lymphoblasts that have been busted open during the preparation of the slide. Lymphoid cells are more fragile than myeloid cells, and often they will burst as the blood (or bone marrow) is spread across the slide. These ghost cells can be seen in any lymphoid process (even a benign lymphocytosis), so they don’t help in differentiating between a benign and a malignant process; but they do help a bit in differentiating between myeloid and lymphoid. The presence of these cells is a pretty soft sign though. You’d want to do markers to make a definitive diagnosis. Cytochemical stains There aren’t any cytochemical stains for lymphoblasts. You can do an MPO or NSE to look for a myeloid process. If the MPO and/or NSE are positive, you know it’s AML. But remember, the blasts in AML-M0 and AML-M7 are generally negative for MPO and NSE! So even with a negative MPO and NSE, it still might be AML. Immunophenotyping Immunophenotyping is a helpful tool for diagnosing ALL. You can’t tell what the blasts are by looking at them under the microscope (except for Burkitt leukemia cells, which look bizarre), and cytochemical staining may be suggestive (but not definitive). So you really need to see what markers are on the surface of the cells. You need this information first of all to tell if it’s an ALL (as opposed to an AML), and second to classify the ALL. Usually, the flow lab will have an ALL panel that they do for every potential case of ALL. It includes B-cell markers (like CD21), T-cell markers (like CD3), TdT (produced in the nucleus) and immunoglobulin (this can be either in the cytoplasm or on the cell surface). Some cases of ALL will also show aberrant expression of myeloid markers; but it’s usually easy to tell that it’s ALL because most of the markers are lymphoid.

www.pathologystudent.com

The Complete (but not obsessive) Hematopathology Guide page 62

Cytogenetics/Molecular Studies Two cytogenetic abnormalities are worth noting here: t(9;22) and t(8;14). Some cases of precursor B ALL have the Philadelphia chromosome (the mutated chromosome 22 in the t(9;22) translocation). This is strange, because the Philadelphia chromosome is a characteristic finding in chronic myeloid leukemia. This translocation is more likely to show up in cases of ALL occurring in adults, and it is a poor prognostic indicator. The t(8;14) translocation is characteristic of Burkitt leukemia. As mentioned above, the translocation involves the c-myc gene on chromosome 8 and the heavy chain locus on chromosome 14. The c-myc gene product activates transcription (in other words, it makes the cell grow). If you put the c-myc gene next to the heavy chain gene (which is always being transcribed), then you’ve set up the cell to grow like crazy. Sometimes, instead of winding up next to the heavy chain locus on chromosome 14, c-myc moves next to one of the light chain genes (the kappa gene is on 2 and the lambda gene is on 22). The same thing happens in these t(2;8) and t(8:22) translocations: c-myc gets transcribed like crazy, making the cell grow like crazy.

Treatment and Prognosis There are several factors that you can look at to help determine prognosis in a patient with ALL: • • • •

Immunophenotype (best prognosis: precursor B ALL) Age (best prognosis: patients between 1 and 10 years of age) WBC (best prognosis: less than 10 x 109/L) Cytogenetics (best prognosis: normal or hyperdiploid tumor cells)

ALL prognosis in kids is good.

Treatment in ALL is similar to that in AML, and involves chemotherapy with or without bone marrow transplantation. Fortunately, the prognosis is relatively good for most children with ALL; with aggressive chemotherapy, about 80% are cured. Adults do not fare as well; only about 40% of adults with ALL are cured of their disease.

www.pathologystudent.com

The Complete (but not obsessive) Hematopathology Guide page 63

Chronic Leukemias Chronic leukemias are very different from acute leukemias. Chronic leukemias are for the most part diseases of older adults (acute leukemias occur in both children and adults). They appear in an insidious fashion and have a relatively good prognosis (as opposed to acute leukemias, which have a stormy onset and poor prognosis). In addition, chronic leukemias are composed of fairly mature-appearing hematopoietic cells (as opposed to acute leukemias, which are composed of blasts). There are two kinds of chronic leukemias: myeloid and lymphoid. Instead of being reasonable, and calling them “chronic myeloid leukemias” and “chronic lymphoid leukemias,” the powers that be dubbed the two divisions “chronic myeloproliferative disorders” and “chronic lymphoproliferative disorders.” These names are not so great, in my opinion, since these are not just “disorders” – they are real leukemias! But no one asked me.

Pathophysiology The chronic leukemias are malignant, monoclonal proliferations of mostly mature myeloid or lymphoid cells in the bone marrow (and blood). These leukemias progress more slowly than acute leukemias. So early on, the marrow is involved – but not totally replaced – by malignant cells. Still, it is hard for the normal white cells to function properly. The lymphoid cells, in particular, have a hard time making normal immunoglobulin in certain chronic lymphoproliferative disorders. One of the major causes of mortality in these patients is infection. As the chronic leukemias evolve, more and more of the marrow is replaced by tumor, and eventually there is little room for normal white cells to grow.

Clinical Features Chronic leukemias present in over a period of weeks or months. Patients might have splenomegaly (which shows up as a dragging sensation or fullness in the left upper quadrant of the abdomen), lymphadenopathy, or a general feeling of malaise and fatigue. Some patients are asymptomatic at diagnosis, and the disease is picked up on a routine blood smear or CBC. Likewise, the clinical course is different in chronic leukemia. In many cases of chronic leukemia, patients can live for years without treatment at all.

www.pathologystudent.com

The Complete (but not obsessive) Hematopathology Guide page 64

Chronic Myeloproliferative Disorders The chronic myeloproliferative disorders are malignant clonal proliferations of a pluripotent stem cell that lead to excessive proliferation of myeloid cells in the blood and bone marrow. What that means in plain English is that a stem cell somewhere way back (before it’s even committed to the neutrophil line, or red cell line) goes bad and starts proliferating like crazy – so you wind up with a marrow packed with cells from all the myeloid lineages (the official name is “panhyperplasia”). Usually, one particular myeloid lineage predominates in this growth fest – so you’ll see a ton of all the myeloid cells, but the majority are neutrophils, or red cells, or megakaryocytes. So the chronic myeloproliferative disorders have been divided into four types according to what is proliferating most: • • • •

Chronic myeloid leukemia (tons of neutrophils and precursors) Polycythemia vera (tons of red cells and precursors) Essential thrombocythemia (tons of platelets and megakaryocytes) Chronic myelofibrosis (tons of everything…then nothing! See below.)

Chronic myeloproliferative disorders:

CML, PV, ET, and chronic myelofibrosis

We’ll consider each of these separately because they are very different clinically and morphologically. But they do have some common features: all of them have a high white count with a left shift, a hypercellular marrow, and splenomegaly.

Chronic myeloid leukemia Chronic myeloid leukemia (CML) is a chronic myeloproliferative disorder characterized by a marked proliferation of neutrophils (and precursors) in the bone marrow and blood. All cases have a t(9;22), also known as the Philadelphia chromosome (it’s the 22 that’s officially the Philadelphia chromosome).

CML has a t(9;22).

Clinical Features CML frequently occurs in patients who are around 40 or 50. It does not occur in children (though there is a separate disease similar to CML, called juvenile CML, that does occur in kids). Usually, the onset is slow, with a long asymptomatic period, followed by fevers, fatigue, night sweats and abdominal fullness. On physical exam, patients usually have an enlarged spleen. Hepatomegaly and lymphadenopathy may also be present.

www.pathologystudent.com

The Complete (but not obsessive) Hematopathology Guide page 65

There are three clinical stages, or phases, of CML: chronic phase, accelerated phase and blast crisis. Patients generally present in chronic phase and then progress to one or both of the other phases. Chronic phase • High but stable number of neutrophils and precursors. • Stable hemoglobin and platelet count. • Easily controlled by therapy. • With traditional treatment (not imatinib, see below), usually lasts 3-4 years; is then followed by accelerated phase and/or blast crisis. Accelerated phase • Characterized by a change in the patient's previously stable state. • Usually see increasing leukocytosis, decreasing hemoglobin and platelet count. • May terminate in this stage, or may progress to blast crisis. • Usually fatal within several months.

Chronic myeloid leukemia:

marked neutrophilia,

left shift, and basophilia

Blast crisis • Characterized by a marked increase in blasts (myeloblasts or lymphoblasts). • Usually fatal within a few weeks or months. Morphology Blood The blood smear shows a marked neutrophilia with a left shift. The left shift is a little weird in that it is not evenly distributed between all the neutrophil stages. There are tons of neutrophils at all stages of development, but there are relatively more myelocytes and segmented neutrophils (and relatively less of the other stages). There are a few myeloblasts around (which you don’t see in normal blood, of course) but they don’t number more than 2 or 3%. Here’s an interesting thing: patients with CML almost always have a basophilia. That’s actually one of the first things that happens in the development of the disease! There are few if any other reasons for a basophilia. So if you see this in a patient, even if they don’t have the typical findings of CML (big white count with lots of neutrophils and precursors), you should rule out CML! The platelet count may be increased (because of all the megakaryocytes around in the bone marrow).

www.pathologystudent.com

CML: blood

The Complete (but not obsessive) Hematopathology Guide page 66

Bone marrow The bone marrow is hypercellular, with a pan-myeloid hyperplasia (all the myeloid cells are increased – neutrophils and precursors, red cell precursors, megakaryocytes). However, if you look closely, you’ll see that the neutrophils and precursors make up the bulk of the cells. Later in the course of the disease, the marrow may become fibrotic. You can detect this using a reticulin stain. This is not a good sign. Pathophysiology All cases of CML have a translocation between chromosomes 9 and 22, resulting in what’s commonly known as the Philadelphia chromosome (Ph). This designation refers to the new chromosome 22 that results from the translocation. Nobody talks about poor chromosome 9. The translocation places the c-abl proto-oncogene on chromosome 9 next to the bcr gene on chromosome 22. A new, fusion gene is created: the bcr-abl gene. The bcr-abl gene encodes a protein called p210, which increases tyrosine kinase activity and disrupts the cell cycle. Here’s a weird fact: the Philadelphia chromosome is found not only in the myeloid cells, but also in some B lymphocytes! That’s weird, considering that this is a myeloid lesion with no apparent changes in the lymphoid cells. This probably means that the initial bad cell (the one that became malignant) was a very early stem cell, one that hadn’t even committed itself to myeloid or lymphoid lineage yet, and the Philadelphia chromosome is present in all the descendents of that cell. Further supporting this idea is the fact that when patients enter blast crisis, the blasts can be lymphoid!

CML: bone marrow

Treatment and Prognosis In the old days, CML was treated with myelosuppressive agents like hydroxyurea, and then if the patient had a match and could tolerate it, allogeneic bone marrow transplant was performed. That was the only hope for a cure. Recently, a new drug called imatinib (or Gleevec) was developed that targets the messed-up tyrosine kinase receptor activity in CML. It has been like a miracle for many patients – even patients in the later stages of the disease. In fact, we don’t even know what the typical prognosis of CML is anymore, because these patients are still living with the disease. This drug has turned CML into a chronic but treatable disease, like diabetes, for many patients. It’s one of the happiest leukemia research stories ever.

www.pathologystudent.com

The Complete (but not obsessive) Hematopathology Guide page 67

Chronic myelofibrosis Chronic myelofibrosis, also called idiopathic myelofibrosis or agnogenic myeloid metaplasia, is a chronic myeloproliferative disorder characterized by panmyelosis, bone marrow fibrosis, and extramedullary hematopoiesis. In real words: the bone marrow at first is markedly hypercellular, but over time, it becomes fibrotic, and the hematopoietic cells go elsewhere (most often, to the spleen) to try to make a home. Clinical Features Like the other chronic leukemias, chronic myelofibrosis is a disease of older adults. The peak age is in the late 50s. The disease presents over a relatively long period of time, with symptoms of splenomegaly (left upper quadrant pain and fullness, epigastric pressure) and anemia (weakness, fatigue, and palpitations). A small number of patients are asymptomatic at diagnosis. Physical examination shows massive splenomegaly in most patients, as well as signs of anemia (pallor, tachycardia).

Chronic myelofibrosis: blood

Chronic myelofibrosis:

Morphology Blood The blood smear shows a leukoerythroblastosis (remember this term from benign leukocytoses? If not, see page 41). There are lots of teardrop red cells in the blood due to the tight spaces (fibrotic marrow, big spleen) the red cells have to squeeze through. The platelets are often weird looking (large and hypogranular).

marrow fibrosis,

extramedullary hematopoiesis and teardrop red cells

Bone marrow Early on, the marrow is hypercellular, with a pan-myeloid hyperplasia. Megakaryocytes, in particular, are markedly increased in number. Later on, the marrow becomes fibrotic, and in the end stages of the disease, it is entirely replaced by fibrotic tissue, with very few remaining hematopoietic cells. Pathophysiology The cause of the fibrosis is still not completely worked out. The fibroblasts are benign – so why are they so active? It’s probably a result of megakaryocyte stimulation. Megakaryocytes are known to release cytokines that stimulate fibrosis – and in chronic myelofibrosis, there are tons of megakaryocytes around. So this seems like a plausible explanation.

www.pathologystudent.com

Chronic myelofibrosis: bone marrow

The Complete (but not obsessive) Hematopathology Guide page 68

Treatm ent and Prognosis Chronic myelofibrosis has a relatively long course (mean survival is 5 years). Treatment usually consists of supportive measures (like red cell transfusions) and myelosuppressive therapy (like hydroxyurea) if the patient can tolerate it. The cause of death in patients with chronic myelofibrosis is usually marrow failure. A small number of patients undergo leukemic transformation (meaning that they develop an acute leukemia – like the blast crisis phase we talked about in CML). Interestingly, the acute leukemia can be either myeloid or lymphoid!

Polycythemia Vera Polycythemia vera (PV) is a chronic myeloproliferative disorder characterized by panmyelosis, with an erythroid predominance. In real words: the marrow is stuffed with myeloid cells, and most of them are red cell precursors. The blood shows a markedly increased red cell count. “Polycythemia" just means an increase in red blood cell mass. It may be: • Primary (polycythemia vera or true polycythemia): increase in red blood cells caused by an intrinsic abnormality of myeloid cells (no # in erythropoietin). • Secondary: increase in red blood cells caused by # secretion of erythropoietin, which may be appropriate (e.g., high-altitude living) or inappropriate (e.g., a paraneoplastic syndrome related to a solid tumor). To distinguish between primary and secondary polycythemia, and to differentiate PV from other chronic myeloproliferative disorders, a polycythemia vera study group came up with the following Polycythemia Vera Study Group Criteria (how creative). To diagnose PV, you need either (1) A1, A2, and A3, or (2) A1, A2, and any two from B: A (m ajor) A1 A2 A3

criteria Increased RBC mass Normal O2 saturation in blood Splenomegaly

B (minor) B1 B2 B3 B4

criteria Thrombocytosis High WBC without infection Increased leukocyte alkaline phosphatase (see box at right) without infection Increased serum B12 level

www.pathologystudent.com

Polycythemia vera: tons of red cells

Things to make you look smart Q. What is leukocyte alkaline phosphatase (LAP)? A. LAP is an enzyme present in normal neutrophils. It is strangely absent in the neutrophils in CML – so it was used in the olden days (before cytogenetics) to distinguish between a benign neutrophilia and CML. LAP is expressed at normal levels in the neutrophils in PV. Why is that? Who knows! But it’s important – because if you have a lot of neutrophils around, and you think it’s a chronic myeloproliferative disorder, you could do an LAP. If it was low or zero, that would be a pretty good indicator of CML. If it was increased, it could be PV (you’d still have to rule out infection though). Nowadays, we use other means to diagnose these disorders. But you’ll still hear people (and books) talk about the LAP. So now you know.

The Complete (but not obsessive) Hematopathology Guide page 69

Clinical Features Polycythemia vera, like the other chronic leukemias, is a disease of older adults; the mean age at diagnosis is 60. Symptoms and signs are related to the massive increase in red cell mass. Patients may have headaches, weakness, pruritis and dizziness (from increased blood volume); they may also have symptoms of vascular stasis, thrombosis or infarction (from increased blood viscosity). Physical examination may show hepatosplenomegaly, and something called “plethora,” which means ruddiness or redness of the head and neck. Morphology Blood For most of the course of the disease, the red cell count is markedly increased. So are the white cell count and the platelet count. Towards the end stages of the disease, though, the marrow can become pooped out (the official name for this is “spent phase”) and quit making red cells. Then the patient’s red cell count goes down, and the patient may even become anemic. Bone marrow The marrow is hypercellular, with a pan-myeloid hyperplasia. Red cells make up the bulk of the myeloid cells in the marrow. Towards the end of the disease, however, the marrow may become fibrotic, and red cell production may decrease. Pathophysiology Recently, a unique genetic abnormality was found to be present in virtually all patients with polycythemia vera. There is a normal signaling pathway present in all kinds of organisms, from slime molds to humans, called the JAK-STAT (Janus kinase-signal transducer and activator of transcription) pathway. It’s a pretty cool pathway because it transmits signals from outside of the cells (like growth hormone signals) to the nucleus of the cell without the need for second messengers (which most other receptors need to use). It turns out that myeloid growth and development is mediated, in part, by this pathway. Patients with polycythemia vera have a mutation in the JAK part of this pathway (specifically, in the JAK-2 gene), which makes the JAK think it’s getting signals when it’s not. So cells with this mutation are constantly getting signals to grow! The mutation has been found in virtually all cases of polycythemia vera (making it a great tool for differentiating primary from secondary polycythemia), and in a significant number of patients with chronic myelofibrosis and essential thrombocythemia too.

Virtually all cases of polycythemia vera have a JAK-2 mutation

Treatment and Prognosis Treatment usually involves phlebotomy, with or without myelosuppressive drugs. Survival is long (average 9 - 14 years). Dangers include thrombosis, hemorrhage, and transformation into acute leukemia.

www.pathologystudent.com

The Complete (but not obsessive) Hematopathology Guide page 70

Essential Thrombocythemia Essential thrombocythemia (ET) is a chronic myeloproliferative disorder characterized by panmyelosis, with a megakaryocytic predominance. Meaning: the marrow is stuffed full of myeloid cells, and the predominating cell is the megakaryocyte. The blood shows a markedly increased platelet count.

Essential thrombocythemia: tons of platelets

The diagnosis of ET is basically one of exclusion. You have to rule out benign causes of thrombocytosis and all of the other chronic myeloproliferative disorders. Here are the criteria: • Platelet count must be >600 x 109/L (normal = 150 – 450). • Hgb must be 40). That’s important to know, because this disease looks for all the world like a benign, mature lymphocytosis (like one you’d see in infectious lymphocytosis). The patient’s age can be a very important piece of information when you’re trying to make a diagnosis.

CLL = SLL

Patients with CLL are often asymptomatic for quite some time. The disease may be picked up incidentally on a CBC done for some other reason. When symptoms do occur, they are related to organ infiltration (lymphadenopathy, hepatosplenomegaly) or to infection (some patients with CLL have a hypogammaglobulinemia, which makes it difficult to fight off infections). Remember how we talked about lymphoma/leukemia pairs? We already discussed a few: precursor T-lymphoblastic leukemia/lymphoma, precursor B-lymphoblastic leukemia/lymphoma,

www.pathologystudent.com

The Complete (but not obsessive) Hematopathology Guide page 72

and Burkitt leukemia/lymphoma. Well, here’s another one: regular old B-cell CLL is the same thing as a lymphoma called small lymphocytic lymphoma (they are now lumped together under the name B-cell CLL/SLL). It’s actually kind of nice for students, because it cuts down the number of diseases to memorize! Learn about CLL and you already know about SLL. Morphology Blood The blood smear shows a proliferation of mature-appearing lymphocytes (small cells, with clumped/smudged chromatin). The white count may be massively elevated, or just slightly above normal. Bone marrow Proliferation of mature lymphocytes. The more marrow involved, the worse the prognosis. Immunophenotype Immunophenotyping is an important part of diagnosing any of these chronic lymphoproliferative lesions. Some of them can look quite similar, or can look like other lymphoid malignancies – and the only way you can really tell them apart is by doing immunophenotyping.

CLL: blood

Regular old CLL (not the rare T-cell type) has a weird immunophenotype that makes it easy to diagnose. It is: • • • •

negative for TdT (that’s what’s you’d expect; TdT is an enzyme found in lymphoblasts) positive for the B-cell antigens CD19, CD20, CD23 (yeah, okay, it’s a B-cell neoplasm) positive for monoclonal surface Ig (sounds good, B cells make immunoglobulin) positive for the T-cell antigen CD5 (whoa, wait a minute!)

So the CLL cell looks just like a maturing B cell, but it has aberrant expression of CD5 (a marker you normally associate with T cells). It turns out that for a very short period of time during maturation, B cells express CD5 (but then lose it again). So maybe that’s where the CD5 expression in CLL comes from.

CLL is a B-cell neoplasm with aberrant expression of CD5.

Pathophysiology A small number of cases show rearrangement of bcl-2, a protein involved in apoptosis. This rearrangement leads to over-expression of bcl-2, prevention of apoptosis, and cell immortality. Which kinda fits what’s going on clinically in patients with CLL – the cells don’t grow explosively (like the cells in Burkitt lymphoma), they just sort of don’t die off. The most common chromosomal abnormalities is trisomy 12 (which confers a bad prognosis).

www.pathologystudent.com

The Complete (but not obsessive) Hematopathology Guide page 73

Treatment and Prognosis Patients with CLL are not really treated until they become symptomatic. The treatment, at least currently, is often worse than just existing with the disease (and it doesn’t prolong survival). When treatment is given, it usually consists of corticosteroids and chemotherapy. Bone marrow transplant is not useful. There are a few staging systems that help predict survival in CLL. The Rai and Binet systems are the most common. Both use several variables (like white count, lymphadenopathy, hepatosplenomegaly, anemia, and thrombocytopenia) to predict median survival. The most important adverse variables appear to be anemia and thrombocytopenia, surprisingly (you’d expect it to be the white count!). The amount and pattern of bone marrow involvement is also helpful in determining prognosis; the more marrow involvement there is, and the more diffuse (as opposed to patchy) the pattern is, the worse the prognosis is. The prognosis varies quite a bit from patient to patient (some patients live many years, others have rapid course), but overall, the prognosis is pretty good (the mean is somewhere around 10 years). If a patient dies of the disease, it is usually due to infection. Some patients develop an aggressive large-cell lymphoma (this is called “Richter’s transformation”) which usually heralds a terminal phase of the disease.

Hairy Cell Leukemia Hairy cell leukemia (HCL) is a rare chronic lymphoproliferative disorder of B-cell origin in which the cells look “hairy.” The primary sites involved are blood, bone marrow, and spleen. Clinical Features HCL usually affects older patients (40-60), and it has a weird male predominance (M:F – 5:1). Patients usually have a huge spleen. Funny, though – they usually don’t have any lymphadenopathy (usually, if you have a lymphoid neoplasm with a big spleen, you’ll have lymphadenopathy too). Morphology Blood Patients with HCL are often pancytopenic. (That’s weird – usually patients with leukemia have a high white count!) We’ll see why that is in a minute. The few leukemic cells that are present have a shaggy appearance – the cytoplasm appears to have kind of a messy, blurry border. That’s because the cells actually have very fine cytoplasmic projections. If you look under an electron microscope, they really look like hairs! Another unusual feature: monocytopenia is always present! Even in cases without a pancytopenia. Why is that? Who knows.

www.pathologystudent.com

HCL: pancytopenia (or at least monocytopenia)

The Complete (but not obsessive) Hematopathology Guide page 74

Bone marrow The bone marrow has an unusual appearance. It’s hypercellular (meaning there aren’t many fat spaces), but loosely structured (it’s said to have a "moth-eaten" appearance). In part, this is due to the characteristics of the hairy cells: they have quite a bit of cytoplasm, and these hairy things too, which tend to push the cells apart from each other. So when you look at a bunch of hairy cells, the nuclei aren’t closely smushed together (like CLL/SLL cells would be), but spaced apart, imparting a loose appearance at low power. Another factor leading to the “moth-eaten” appearance is reticulin fibrosis. For some reason, the hairy cells elicit a fibrotic response in the marrow, and you get little strands of reticulin surrounding each cell, kinda like chicken wire. This makes it pretty difficult for the cells to get out, too, if you think about it – so that’s why the blood and bone marrow aspirate are pancytopenic. Cytochemistry Here’s one of the few times that cytochemical stains are of use in lymphoid neoplasms. There’s a special stain called tartrate-resistant acid phosphatase (or TRAP) that’s positive in hairy cells (but not in other small lymphocytes). What you do is stain the slide with acid phosphatase (which stains a whole bunch of things red, like neutrophils, monocytes, and platelets), and then wash it with tartrate (which washes the red out of almost every cell except hairy cells).

HCL: hairy cell (top) and normal lymphocyte (bottom)

Immunophenotype Here’s the immunophenotype for hairy cell leukemia: • • • •

positive for B-cell antigens (CD19, CD20) positive for CD25 (a marker that you normally see on monocytes and T cells) positive for CD11c (also on monocytes and T-cells) negative for CD5 (the T-cell antigen expressed in CLL).

Treatment and Prognosis In the olden days, the only treatment for HCL was splenectomy and observation (with a relatively grave prognosis). Now, using new chemotherapeutic agents (2-chlorodeoxyadenosine, pentostatin, or interferon-alpha), the 5-year survival rate is over 85%.

www.pathologystudent.com

Hairy cells positive for TRAP

The Complete (but not obsessive) Hematopathology Guide page 75

Prolymphocytic leukemia Prolymphocytic leukemia (PL) is a rare chronic lymphoproliferative disorder in which the predominant cell is the prolymphocyte. Most cases of PL are of B-cell type, but a minority are of Tcell type. Clinical Features PL generally occurs in older patients. It usually presents with systemic disease, including splenomegaly. It is more aggressive than the other chronic lymphoproliferative disorders.

PL is more aggressive than other chronic lymphoproliferative disorders

Morphology Blood The blood smear shows a proliferation of lymphocytes, over half of which are prolymphocytes. Prolymphocytes are cells that you normally don’t see in the blood or bone marrow. They’re large cells with coarse chromatin and a single, prominent nucleolus. Bone marrow The bone marrow shows a proliferation of the same type of cells. Immunophenotype Cases of PL that are of B-cell lineage show the following immunophenotype: • positive for B-cell antigens (CD20, CD22) • usually negative for CD5 (but sometimes positive) T-lineage PL shows the immunophenotype of a mature T lymphocyte: • positive for T-cell antigens (CD2, CD3, CD7) • negative for TdT • Many cases are CD4+/CD8-; a smaller number are CD4+/CD8+ or CD4-/CD8+ Treatment and Prognosis Both T-cell and B-cell PL are aggressive disorders. The B-cell type is somewhat responsive to standard chemotherapy, but the survival is still short. The T-cell type is pretty much unresponsive to standard chemotherapy, but some newer agents (like rituximab) seem to work a little better. Survival is still dismal though.

www.pathologystudent.com

The Complete (but not obsessive) Hematopathology Guide page 76

Large Granulated Lymphocyte Leukemia Large granulated lymphocyte leukemia (LGLL) is a rare chronic lymphoproliferative disorder of T cells which are large and display cytoplasmic granules. There are two subtypes: T-cell type and natural killer cell type. Clinical Features LGLL usually occurs in elderly age ranges. The T-cell type of LGLL is an indolent disease, with symptoms mostly related to severe neutropenia (recurrent infections are common). The NK type, in contrast, is a very aggressive disease. Morphology Blood The blood smear shows a lot of large lymphocytes with abundant, clear cytoplasm and cytoplasmic granules. The neutrophil count is decreased – sometimes markedly so. Bone marrow The bone marrow shows the same population of malignant cells as the blood, as well as the same decrease in neutrophils. Immunophenotype The T-cell type of LGLL shows the following immunophenotype: • positive for CD2, CD3, CD8 and CD57 • negative for CD4 The NK type of LGL shows the following immunophenotype: • positive for CD2 and CD56 • negative for CD3, CD4 and CD8 Treatment and prognosis Patients with the T-cell type respond well to low-dose traditional chemotherapy, and have longlasting remissions, with a mean survival of 10 years. The NK type, however, does not respond to chemotherapy, and has a short survival.

www.pathologystudent.com

The Complete (but not obsessive) Hematopathology Guide page 77

5. Multiple Myeloma Multiple myeloma is a malignant, clonal disorder of plasma cells which originates in the bone marrow. Three key features are present: 1. A monoclonal proliferation of plasma cells 2. A monoclonal gammopathy 3. Decreased levels of other, normal (polyclonal) immunoglobulins.

Clinical Features Myeloma is a relatively common disorder (1% of all malignancies and 10% of all hematologic malignancies in adults). It occurs in adults (rare #!C+6(/,(;&.! D#!C+65*('4&*! E#!@+5.!'+,,!%+6(/,(;&.!'(.'+.*45*&(.!C>?!! $%&'%!()!*%+!)(,,(-&./!&.0&'+1!*+,,1!2(3!-%+*%+4!5!75*&+.*G1!4+0!'+,,1!54+!6&'4('2*&'H!.(46('2*&'H!(4!65'4('2*&'8!! 9#!@+5.!'+,,!%+6(/,(;&.!'(.'+.*45*&(.!C>?! =#!:+0!;,((0!'+,,!'(3.*!?! >#!C+65*('4&*! D#!C+6(/,(;&.! E#!@+5.!'+,,!A(,36+!B?!

! I#!! 9.&1('2*(1&1!4+)+41!*(J! 9#!K4(,&)+45*&(.!()!4+0!'+,,1!! =#!9;1+.'+!()!4+0!'+,,1! >#!B&4*35,!5;1+.'+!()!%+6(/,(;&.!&.!4+0!'+,,1! D#!B54&5*&(.!&.!4+0!'+,,!1&L+!! E#!B54&5*&(.!&.!4+0!'+,,!1%57+! ! !

www.pathologystudent.com

The Complete (but not obsessive) Hematopathology Guide page 102

Anemia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

9#!9.+6&5!()!'%4(.&'!0&1+51+! =#!$546!53*(&663.+!%+6(,2*&'!5.+6&5!! >#!>(,0!53*(&663.+!%+6(,2*&'!5.+6&5! D#!C+4+0&*542!17%+4('2*(1&1! E#!R,3'(1+NSN7%(17%5*+N0+%204(/+.51+!0+)&'&+.'2!

! ! ! ! ! U#!!

9#!D+'4+51+0!;&,&43;&.! =#!D+'4+51+0!TDC! >#!D+'4+51+0!%57*(/,(;&.! D#!D+'4+51+0!4+*&'3,('2*+1!

! ! ! ! ! !

9#!9.+6&5!()!'%4(.&'!0&1+51+! =#!$546!53*(&663.+!%+6(,2*&'!5.+6&5!! >#!>(,0!53*(&663.+!%+6(,2*&'!5.+6&5! D#!C+4+0&*542!17%+4('2*(1&1! E#!R,3'(1+NSN7%(17%5*+N0+%204(/+.51+!0+)&'&+.'2!

$%&'%!()!*%+!)(,,(-&./!,5;!4+13,*1!&1!'(.1&1*+.*!-&*%!5!%+6(,2*&'!74('+118!

$%&'%!*27+!()!5.+6&5!&.A(,A+1!*%+!5**5'%6+.*!()!Q/@!5.0!'(67,+6+.*!*(!4+0!'+,,18!

www.pathologystudent.com

The Complete (but not obsessive) Hematopathology Guide page 103

V#!! P%+!4+0!'+,,1!&.!1&'O,+!'+,,!0&1+51+!54+!6(4+!131'+7*&;,+!*(!%+6(,21&1!;+'531+J! ! 9#!P%+!4+0!'+,,!6+6;45.+!&1!0+)+'*&A+! ! =#!P%+2!%5A+!,+11!%+6(/,(;&.! ! >#!P%+2!,5'O!5!'+,,N1*5;&,&L&./!+.L26+!! ! D#!P%+2!54+!+17+'&5,,2!23662!*(!65'4(7%5/+1! ! E#!P%+2!7(11+11!5.!5;.(465,!%+6(/,(;&.!-%&'%!7(,26+4&L+1!37(.!0+(W2/+.5*&(.!! ! S#!! X(34! FMN2+54N(,0H! %+5,*%2! 1&1*+4! -51! )(3.0! *(! %5A+! 5! ,(-! %+6(/,(;&.! -%+.! 1%+! *4&+0! *(! 0(.5*+! ;,((0#!C+4!0('*(4!(40+4+0!5!>=>H!-%&'%!1%(-+0!*%+!)(,,(-&./J! ! !

C/;!!"Y!/Z0T!B![V!)T!! E#!B&*56&.!D!

www.pathologystudent.com

The Complete (but not obsessive) Hematopathology Guide page 104

M#!! a.+! ()! 2(34! 75*&+.*1H! 5! SVN2+54N(,0! )+65,+H! %51! ;++.! %5A&./! 74(/4+11&A+,2! -(41+.&./! )5*&/3+! 5.0! 1%(4*.+11!()!;4+5*%!(A+4!*%+!751*!)+-!6(.*%1#!b%+!5,1(!%51!%50!5!)+-!+7&1(0+1!()!7.+36(.&5H!(.+!()! -%&'%!,5.0+0!%+4!&.!*%+!%(17&*5,H!5.0!1%+!%51!0&))31+!7+*+'%&5+!1'5**+4+0!(A+4!%+4!5461!5.0!.+'O#!C+4! >=>!1%(-1!*%+!)(,,(-&./J! ! !

C/;!!S!/Z0T!B!\Y!)T!#!C+&.L!;(0&+1! D#!:+0!;,((0!'+,,!5//,3*&.5*&(.! E#!b&'O,+!'+,,1! !

www.pathologystudent.com

The Complete (but not obsessive) Hematopathology Guide page 105

Benign leukocytoses ! ! "#!$%&'%!()!*%+!)(,,(-&./!&1!*43+!4+/540&./!*%+!&663.(7%+.(*27+!()!.(465,!7+4&7%+45,!;,((08! 9#!=!'+,,1!5.0!P!'+,,1!54+!74+1+.*!&.!4(3/%,2!*%+!156+!74(7(4*&(.! =#!=!'+,,1H!P!'+,,1!5.0!c`!'+,,1!54+!74+1+.*!&.!4(3/%,2!*%+!156+!74(7(4*&(.! >#!@(1*!()!*%+!,267%('2*+1!54+!c`!'+,,1! D#!@(1*!()!*%+!,267%('2*+1!54+!=!'+,,1! E#!@(1*!()!*%+!,267%('2*+1!54+!P!'+,,1! ! F#!$%&'%!()!*%+!)(,,(-&./!;,((0!)&.0&./1!&1!1++.!6(1*!'(66(.,2!&.!&.)+'*&(31!6(.(.3',+(1&18! 9#!@5*34+!,267%('2*(1&1! ! =#!:+5'*&A+!,267%('2*(1&1!! >#!@5*34+!.+3*4(7%&,&5!! D#!T+)*!1%&)*! E#!T+3O(+42*%4(;,51*(*&'!4+5'*&(.! ! I#! X(34! 75*&+.*! %51! 5.! +,+A5*+0! .+3*4(7%&,! '(3.*#! P%+! ;,((0! 16+54! 1%(-1! ,(*1! ()! 3.3135,N,((O&./! .+3*4(7%&,1d!1(6+!%5A+!5;3.05.*H!054ON7347,+!'2*(7,516&'!/45.3,+1H!5.0!(*%+41!%5A+!,(*1!()!A5'3(,+1!&.! *%+!'2*(7,516#!$%&'%!()!*%+!)(,,(-&./!0&1(40+41!&1!*%+!6(1*!,&O+,2!'531+!()!*%+1+!)&.0&./18! ! 9#!E#!'(,&!1+71&1!! ! =#!T3731! ! >#!b*4+11! ! D#!D43/!4+5'*&(.! ! E#!@2'(1&1!)3./(&0+1ZbeL542!12.04(6+! ! U#!X(34!.+W*!75*&+.*!%51!5.!+,+A5*+0!+(1&.(7%&,!'(3.*#!$%&'%!()!*%+!)(,,(-&./!0&1(40+41!&1!*%+!6(1*!,&O+,2! '531+8! ! ! ! ! !

9#!E#!'(,&!1+71&1!! =#!T3731! >#!b*4+11! D#!D43/!4+5'*&(.! E#!>%4(.&'!62+,()&;4(1&1!

www.pathologystudent.com

The Complete (but not obsessive) Hematopathology Guide page 106

V#!Q*G1!%540!*(!;+,&+A+H!;3*!2(34!.+W*!75*&+.*!%51!5.!+,+A5*+0!;51(7%&,!'(3.*#!b%+!1%(3,0!;+!+A5,35*+0!*(! 1++!&)!1%+!%51J! ! ! ! ! ! !

9#!E#!'(,&!1+71&1!! =#!T3731! >#!b*4+11! D#!D43/!4+5'*&(.! E#!>%4(.&'!62+,(&0!,+3O+6&5!!

Leukemia ! ! "#!$%&'%!()!*%+!)(,,(-&./!&1!*43+!()!5'3*+!,+3O+6&58! !

9#!b267*(61!'(6+!(.!1,(-,2H!(A+4!5!7+4&(0!()!6(.*%1!(4!2+541#! =#!P%+!;(.+!6544(-!5.0!;,((0!54+!)3,,!()!65*34+!,267%(&0!(4!62+,(&0!'+,,1#! >#!Q*!5,6(1*!.+A+4!(''341!&.!503,*1#!! D#!Q)!5!'%&,0!/+*1!5'3*+!,+3O+6&5H!&*!&1!6(1*!()*+.!()!*%+!62+,(&0!,&.+5/+#! E#!Q*!&1!457&0,2!)5*5,!&)!.(*!*4+5*+0!&66+0&5*+,2!

! ! F#!9,,!()!*%+!)(,,(-&./!1*5*+6+.*1!5;(3*!,+3O+6&5!54+!*43+!"#$"%&J! ! !

9#!9'3*+!,+3O+6&51!*27&'5,,2!74+1+.*!-&*%!1300+.N(.1+*!;(.+!6544(-!)5&,34+#! =#!9'3*+!,+3O+6&51!54+!'(67(1+0!()!;,51*1H!5.0!'%4(.&'!,+3O+6&51!54+!'(67(1+0!()!! !!!!65*34&./!'+,,1#! >#!9!PN'+,,!&663.(7%+.(*27+!&1!5!7((4!74(/.(1*&'!&.0&'5*(4!&.!5'3*+!,267%(;,51*&'!! !!!!!,+3O+6&5#! D#!9'3*+!,267%(;,51*&'!,+3O+6&5!&1!6(1*!'(66(.!&.!503,*1#!! E#!>%4(.&'!,+3O+6&51!%5A+!5!74(/.(1&1!(.!*%+!(40+4!()!65.2!2+541#!!

! ! I#!K5*&+.*1!-&*%!-%&'%!()!*%+!)(,,(-&./!*27+1!()!,+3O+6&5!54+!5*!%&/%!4&1O!)(4!0&11+6&.5*+0!&.*45A51'3,54! '(5/3,5*&(.!?8! ! ! ! ! !

9#!>%4(.&'!62+,(&0!,+3O+6&5! =#!>%4(.&'!,267%('2*&'!,+3O+6&5! >#!9'3*+!74(62+,('2*&'!,+3O+6&5!! D#!9'3*+!6(.(;,51*&'!,+3O+6&5! E#!9'3*+!+42*%4(;,51*&'!,+3O+6&5!

www.pathologystudent.com

The Complete (but not obsessive) Hematopathology Guide page 107

! U#! X(3! 54+! *4+5*&./! 5! 75*&+.*! -%(! &1! 3.0+4/(&./! *%+4572! )(4! 5'3*+! ,+3O+6&5#! C+! %51! 0&))31+H! 6511&A+! /&./&A5,!+.,54/+6+.*#!$%&'%!()!*%+!)(,,(-&./!'+,,!*27+1!0(+1!%&1!,+3O+6&5!6(1*!,&O+,2!&.A(,A+8! ! 9#!c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f3+.*,2!&.A(,A+1!+W*456+03,,542!1&*+18! 9#!9'3*+!74(62+,('2*&'!,+3O+6&5! =#!9'3*+!6(.(;,51*&'!,+3O+6&5! >#!9'3*+!+42*%4(;,51*&'!,+3O+6&5! D#!9'3*+!6+/5O542(;,51*&'!,+3O+6&5! E#!9'3*+!,267%(;,51*&'!,+3O+6&5! !

www.pathologystudent.com

The Complete (but not obsessive) Hematopathology Guide page 108

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b267*(61!3135,,2!'(6+!(.!1300+.,2H!(A+4!5!7+4&(0!()!1+A+45,!0521!! ! >#!Q*!&1!g31*!51!'(66(.!&.!'%&,04+.!51!&*!&1!&.!503,*1! D#!P%+!;(.+!6544(-!5.0!;,((0!54+!)3,,!()!;,51*1! E#!Q*!&1!457&0,2!)5*5,!&)!.(*!*4+5*+0!&66+0&5*+,2!

!

! ""#!P%+!K%&,50+,7%&5!'%4(6(1(6+!&1!'%545'*+4&1*&'!()!-%&'%!*27+!()!,+3O+6&58! 9#!9'3*+!62+,(&0!,+3O+6&5! =#!PN'+,,!5'3*+!,267%(;,51*&'!,+3O+6&5! >#!>%4(.&'!,267%('2*&'!,+3O+6&5!! D#!>%4(.&'!62+,(&0!,+3O+6&5!! E#!K(,2'2*%+6&5!A+45! !

www.pathologystudent.com

The Complete (but not obsessive) Hematopathology Guide page 109

"F#! 9! %+5,*%2H! VMN2+54N(,0! 65,+! &1! )(3.0! *(! %5A+! 5! 6&,0! ,267%('2*(1&1! 034&./! 5! 4(3*&.+! 7%21&'5,! +W56&.5*&(.#!9,,!*%+!,267%('2*+1!577+54!165,,!5.0!65*34+H!5.0!&663.(7%+.(*27&./!1%(-1!*%5*!*%+2! +W74+11!;(*%!>D"\!5.0!>DV#!$%5*!&1!*%+!6(1*!,&O+,2!0&5/.(1&1!&.!*%&1!75*&+.*8! 9#!=+.&/.H!4+5'*&A+!,267%('2*(1&1! =#!>%4(.&'!,267%('2*&'!,+3O+6&5Z165,,!,267%('2*&'!,267%(65! >#!=N'+,,!5'3*+!,267%(;,51*&'!,+3O+6&5! D#!PN'+,,!5'3*+!,267%(;,51*&'!,+3O+6&5! E#!@2'(1&1!)3./(&0+1Z!beL542!12.04(6+! ! "I#!9!SYN2+54N(,0!65,+!%51!;,((0!16+54!1%(-&./!5!654O+0!.+3*4(7%&,&'!,+3O('2*(1&1!-&*%!5!,+)*!1%&)*!5.0! ;51(7%&,&5#!C&1!6544(-!&1!%27+4'+,,3,54!5.0!'(67(1+0!74+0(6&.5.*,2!()!.+3*4(7%&,1!5.0!74+'341(41#!9! '2*(/+.+*&'!51152!()!*%+!.+3*4(7%&,1!1%(-1!5!*45.1,('5*&(.!;+*-++.!'%4(6(1(6+1!\!5.0!FF#!$%&'%!()! *%+!)(,,(-&./!1*5*+6+.*1!&1!*43+!4+/540&./!%&1!0&1+51+8! ! ! ! !

9#!Q*!%51!*-(!7%51+1! =#!Q*!&1!5,-521!457&0,2!74(/4+11&A+! >#!Q*!3135,,2!(''341!&.!'%&,04+.! D#!Q*!652!;+!;+.&/.! E#!P%+4+!&1!5!.+-!'%+6(*%+4572!043/!'5,,+0!&65*&.&;!#!a''34!g31*!51!()*+.!&.!'%&,04+.!51!*%+2!0(!&.!503,*1! D#!94+!&.0&1*&./3&1%5;,+!)4(6!+5'%!(*%+4!3.0+4!*%+!6&'4(1'(7+! E#!94+!3135,,2!)5*5,!-&*%&.!5!7+4&(0!()!6(.*%1!

! "V#!9!SYN2+54N(,0!-(65.!74+1+.*1!-&*%!5!%3/+!17,++.!5.0!)5*&/3+#!C+4!;,((0!1%(-1!.36+4(31!*+5404(7N 1%57+0!4+0!'+,,1#!C+4!;(.+!6544(-!&1!)&;4(*&'#!$%5*!&1!*%+!6(1*!,&O+,2!0&5/.(1&18! ! ! ! ! !

9#!9'3*+!62+,(&0!,+3O+6&5! =#!Q.)+'*&(.! >#!b&'O,+N'+,,!5.+6&5! D#!>%4(.&'!62+,()&;4(1&1! E#!90+.('54'&.(65!

www.pathologystudent.com

The Complete (but not obsessive) Hematopathology Guide page 110

!

Myeloma ! ! "#!9,,!()!*%+!)(,,(-&./!54+!*43+!()!63,*&7,+!62+,(65!"#$"%&J! ! 9#!P%+4+!&1!5!65,&/.5.*!74(,&)+45*&(.!()!7,5165!'+,,1!&.!*%+!;(.+!6544(-! ! =#!9!6(.(',(.5,!/566(75*%2!&1!74+1+.*! ! >#!c(465,!&663.(/,(;3,&.1!54+!0+'4+51+0! ! D#!K5*&+.*1!%5A+!(1*+(,2*&'!,+1&(.1! ! E#!EA+.!-&*%(3*!*4+5*6+.*H!*%+!',&.&'5,!'(341+!&1!,(./H!-&*%!5!6+5.!134A&A5,!()!FYi!2+541!! ! F#!$%5*!&1!*%+!6(1*!'(66(.,2N+W74+11+0!%+5A2!'%5&.!&.!63,*&7,+!62+,(65'! ! 9#!Q/9! ! =#!Q/D! ! >#!Q/E! ! D#!Q/R! ! E#!Q/@!

Lymphoma ! ! "#!a.+!()!2(34!75*&+.*1!%50!5!/51*4&'!,267%(65!*%5*!-51!13''+11)3,,2!*4+5*+0!-&*%!5.*&;&(*&'1#!$%&'%!()! *%+!)(,,(-&./!0&0!1%+!6(1*!,&O+,2!%5A+8! ! 9#!b65,,!,267%('2*&'!,267%(65! ! =#!@3'(15N511('&5*+0!,267%(&0!*&113+!#!_(,,&'3,54!'+,,!,267%(65! ! D#!@2'(1&1!)3./(&0+1ZbeL542!12.04(6+! ! E#!T267%(;,51*&'!,267%(65! ! F#!$%&,+!+W56&.&./!5!IVN2+54N(,0!65,+H!2(3!.(*&'+!%+!%51!5!,54/+!,267%!.(0+!&.!%&1!.+'O#!Q*!&1!6(;&,+!5.0! *+.0+4#!P%+!6(1*!,&O+,2!'531+!()!%&1!1267*(61!&1J! ! ! ! !

9#!Q.)+'*&(.! =#!@+*51*5*&'!1f356(31!'+,,!'54'&.(65! >#!=34O&**!,267%(65! D#!C(0/O&.!,267%(65! E#!>%4(.&'!,267%('2*&'!,+3O+6&5Z165,,!,267%('2*&'!,267%(65!

www.pathologystudent.com

The Complete (but not obsessive) Hematopathology Guide page 111

! I#!P&./&;,+N;(02!65'4(7%5/+1!652!;+!1++.!&.J! ! 9#!=34O&**!,267%(65! ! =#!=+.&/.!/+46&.5,!'+.*+41! ! >#!=(*%! ! D#!c+&*%+4! ! U#!9!SMN2+54N(,0!65,+!%51!,54/+H!1'5,2H!4+0!7,5f3+1!(.!%&1!;5'O!5.0!;3**('O1#!9!1O&.!;&(712!1%(-1!165,,! 5;1'+11N,&O+!175'+1!&.!*%+!0+46&1H!)&,,+0!-&*%!'+4+;4&)(46!,267%('2*+1#!C&1!;,((0!'(.*5&.1!*%+!156+!'+,,1H! -%&'%!(.!&663.(7%+.(*27&./!0&17,52!PN'+,,!654O+41#!$%5*!&1!*%+!6(1*!,&O+,2!0&5/.(1&18! ! 9#!b65,,!,267%('2*&'!,267%(65! ! =#!PN'+,,!5'3*+!,267%(;,51*&'!,267%(65! ! >#!D&))31+!,54/+!'+,,!,267%(65! ! D#!@2'(1&1!)3./(&0+1ZbeL542!12.04(6+! ! E#!=34O&**!,267%(65!! ! ! V#!$%&'%!()!*%+!)(,,(-&./!&1!*43+!4+/540&./!C(0/O&.!,267%(658! ! ! ! ! ! ! ! !

9#!Q*!/+.+45,,2!5'*1!6(4+!5//4+11&A+,2!*%5.!.(.NC(0/O&.!,267%(65! =#!P%+!65,&/.5.*!'+,,!&1!*%+!:++0Nb*+4.;+4/!'+,,!! >#!Q*!A&4*35,,2!5,-521!(''341!&.!+,0+4,2!75*&+.*1! D#!Q*!()*+.!74+1+.*1!&.!(.+!,267%!.(0+H!*%+.!j1O&71k!*(!5!0&1*5.*!,267%!.(0+!! E#!b+'(.0!65,&/.5.'&+1!A&4*35,,2!.+A+4!(''34!&.!75*&+.*1!-&*%!C(0/O&.!,267%(65!

www.pathologystudent.com

The Complete (but not obsessive) Hematopathology Guide page 112

Answers to Study Questions Introduction

Leukemia

"#!E! F#!E! I#!D! ! !

"#!E! F#!D! I#!>! U#!=! V#!9! S#!9! [#!=! M#!E! \#!E! "Y#!9! ""#!D! "F#!=! "I#!E! "U#!=! "V#!D! !

Anemia "#!9! F#!=! I#!>! U#!>! V#!E! S#!=! [#!>! M#!D! \#!D! "Y#!9! !

Benign leukocytoses "#!E! F#!=! I#!9! U#!D! V#!E! !

www.pathologystudent.com

Myeloma "#!E! F#!D!

Lymphoma "#!=! F#!9! I#!>! U#!D! V#!=! ! !

The Complete (but not obsessive) Hematopathology Guide page 113

View more...

Comments

Copyright ©2017 KUPDF Inc.
SUPPORT KUPDF